MRCPsych Paper B: 600 MCQs and EMIs Ashok G Patel, Roshelle Ramkisson, Madhavan Seshadri
INDEX
×
Chapter Notes

Save Clear


Mock Exam: 1Chapter 1

 
Questions: MCQs
For each question, select one answer option
 
ORGANISATION AND DELIVERY OF PSYCHIATRIC SERVICES
1. Which one of the following staff and training statements is a standard requirement for ECT treatment, according to the Electroconvulsive Therapy Accreditation Service (ECTAS)?
  1. There should be at least two trained nurses in the treatment area
  2. There should be at least one trained nurse in the recovery area
  3. There can be a different ECT team in clinic every week
  4. The lead ECT nurse develops protocols for ECT prescription
  5. One competent person should be present in cardiopulmonary resuscitation for every two unconscious patients
2. Which one of the following statements about the provision of information on ECT to patients is correct, according to the Electroconvulsive Therapy Accreditation Service (ECTAS)?
  1. Consent can be obtained by the ECT lead nurse
  2. An ‘ECT rights about consent to treatment’ leaflet is provided to detained patients
  3. ECT information should be provided verbally
  4. An explanation on why ECT does not affect cognition should be provided
  5. There is no need to document capacity assessment for informal patients
3. Which one of the following clinical predictors is a response to phototherapy?
  1. Diurnal variation in mood
  2. Increased appetite
  3. Morning slump in energy
  4. Reduced sleep
  5. Winter weight loss
4. You are treating a 34-year-old female patient who suffers from recurrent depressive disorder. She is not responding to conventional medication. She does not want ECT. She came to your clinic today enquiring about rTMS (transcranial magnetic stimulation). Which one of the following statements about rTMS is true?
  1. A small dose of electric current is passed through the brain in rTMS
  2. Patient needs to be anaesthetised during rTMS.
  3. rTMS stands for rhythmic transcranial magnetic stimulation.
  4. The effectiveness of rTMS is measured by the duration of seizures.
  5. The intensity of rTMS is usually set as a percentage of the patient’s motor threshold
25. Which one of the following conditions is a clinical indication for psychosurgery?
  1. Bipolar affective disorder
  2. Bulimia nervosa
  3. Obsessive compulsive disorder
  4. Post-traumatic stress disorder
  5. Schizophrenia
6. Which of the following is considered the best established approach to assess psychiatric needs?
  1. Eysenck Personality Questionnaire
  2. General health questionnaire
  3. Milton Clinical Multi-axial Inventory
  4. Recovery Star
  5. The Camberwell Assessment of Need
7. According to guidance on the Mental Capacity Act (2005) from the Crown Prosecution Service there are three categories of offences against people with a mental disorder. Which one of the following statements is correct?
  1. An offence against someone with a mental disorder who is unable to refuse
  2. An offence against someone in whom there is a suspicion of mental disorder
  3. It is not an offence if sexual activity is with consent in someone with serious mental disorder
  4. Offence against someone with a mental disorder who is unable to refuse
  5. In all these offences, legal definition of mental disorder is different to that in the Mental Health Act 1983 (amended 2007)
8. You are a ST4 Trainee working in drug and alcohol services. The neurologist from the county hospital has referred a 42-year-old man diagnosed with an alcohol-related seizure. You assess him in the clinic. He has no features of alcohol or substance dependence. This is an isolated event and he has never suffered from epilepsy. The patient wants to know whether he can drive his car to work. Which one of the following would you recommned?
  1. Referral to epilepsy guidance related to driving
  2. The patient may continue driving and need not inform the DVLA as this is an isolated alcohol-related seizure.
  3. The patient may continue driving but should inform the DVLA as this is an isolated alcohol-related seizure.
  4. The patient should stop driving for at least 6 months and inform the DVLA as this is an isolated alcohol-related seizure.
  5. Patient should stop driving for at least 3 months and inform the DVLA as this is an isolated alcohol-related seizure.
9. Which one of the following is the only ever mental health-related incident referred to as a ‘never event’ in England?
  1. Death of a patient at the time of receiving ECT.
  2. Death of a patient treated with high-dose antipsychotics.
  3. Inpatient suicide by hanging using a non-collapsible curtain rail.
  4. Serial killing by a patient with schizophrenia.
  5. Suicide by hanging using a non-collapsible curtain rail.
3
 
RESEARCH METHODS, STATISTICS, CRITICAL REVIEW AND EVIDENCE-BASED PRACTICE
Answer questions 10–14 using the following information
A study was carried out to compare an antidepressant A with a control drug B. There were around 200 patients in each group; 190 patients improved on drug A whereas 120 patients improved with drug B (Table 1.1).
Table 1.1   Drug comparison study
Drug
Improved (remission)
Not improved
A (antidepressant)
190
10
B (control)
120
80
10. What is the remission rate of depression for drug A?
  1. 0.2
  2. 0.4
  3. 0.6
  4. 0.95
  5. 1
11. What is the remission rate of depression for drug B?
  1. 0.3
  2. 0.5
  3. 0.6
  4. 0.9
  5. 1.2
12. What is the absolute benefit increase?
  1. 0.15
  2. 0.25
  3. 0.35
  4. 0.45
  5. 0.55
13. What is the number needed to treat (NNT) for drug A?
  1. 1
  2. 3
  3. 5
  4. 10
  5. 20
14. What term is used when the null hypothesis is rejected even though it is true?
  1. Power
  2. Probability
  3. Standard deviation
  4. Type 1 error
  5. 4Type 2 error
For each question, select one answer option
15. What term is used when the null hypothesis is accepted even though it is false?
  1. Accuracy
  2. False positive
  3. Standard error
  4. Type 1 error
  5. Type 2 error
16. What is the average distance from the mean in a normal distribution?
  1. Negative skewed
  2. Positive skewed
  3. Standard deviation (SD)
  4. Standard error
  5. Variable
17. What is an approximate population score within three standard deviations of a normal distribution?
  1. 95.7%
  2. 96.7%
  3. 97.7%
  4. 98.8%
  5. 99.7%
18. What is the ability to detect differences between groups if they are truly present known as?
  1. α
  2. β
  3. Effect size
  4. Power
  5. Probability
19. Which of the following non-parametric tests is used to compare two unpaired groups independently?
  1. Mann–Whitney U test
  2. McNemar’s test
  3. Student’s t-test
  4. Wilcoxon’s rank sum test
  5. χ² test
20. Which of the following parametric statistical tests is used in comparing more than two groups?
  1. Analysis of variance (ANOVA)
  2. Kruskal–Wallis analysis of variance
  3. Paired Student’s t-test
  4. Wilcoxon’s rank sum test
  5. χ² test
21. Which of the following p values (probability) would be considered statistically significant?
  1. ≥0.04
  2. ≥0.06
  3. ≥0.08
  4. 5≥0.10
  5. ≥0.50
22. What is the range for a linear correlation?
  1. −0.01 to + 0.01
  2. −0.05 to + 0.05
  3. −0.1 to + 0.1
  4. −1 to + 1
  5. −1.5 to + 1.5
23. If the mean = 4, standard deviation 1.4 and n =16, what is the 95% confidence interval?
  1. 4 ± 0.15
  2. 4 ± 0.68
  3. 4 ± 2.21
  4. 4 ± 2.31
  5. 4 ± 5.6
24. What is subjective assessment when the item appears to measure the desired qualities?
  1. Face validity
  2. Concurrent validity
  3. Construct validity
  4. Convergent validity
  5. Divergent validity
25. What type of validity assesses the extent to which a depression questionnaire actually measures depression?
  1. Content validity
  2. Construct validity
  3. Convergent validity
  4. Discriminant validity
  5. Predictive validity
26. Which of the following assesses the consistency of items within a scale or subscale?
  1. Interobserver reliability
  2. Internal consistency
  3. Intraclass correlation coefficient
  4. Intraobserver reliability (test–re-test)
  5. κ
27. Which bias occurs in a hospital-based study about the relationship between exposure and disease?
  1. Berkson’s bias
  2. Diagnostic purity bias
  3. Historical control bias
  4. Membership bias
  5. Referral filter bias
28. What type of reliability is measured by an intraclass correlation?
  1. Construct reliability
  2. Internal consistency reliability
  3. Interobserver reliability
  4. 6Split-half reliability
  5. Test–re-test reliability
29. What is the name given to a situation in which the observation is influenced by observer knowledge?
  1. Demand effect
  2. Domino effect
  3. Halo effect
  4. Hawthorne’s effect
  5. Placebo effect
30. What is Neyman’s bias also known as?
  1. Berkson’s bias
  2. Confounding bias
  3. Information bias
  4. Prevalence–incidence bias
  5. Publication bias
31. Which of the following is a critically appraised database?
  1. Clinical Practice Guidelines
  2. Cochrane Library
  3. EMBASE
  4. PsycINFO
  5. PubMed
32. According to current evidence, in which of the following disorders do omega-3 essential fatty acids have a role in treatment?
  1. Affective disorders
  2. Alcohol misuse
  3. Anxiety disorders
  4. Psychotic disorders
  5. Personality disorders
33. A clinical drug study was conducted to evaluate the efficacy of aripiprazole in patients with recurrent depersonalisation–derealisation. It was necessary to calculate the number of participants needed in the study to demonstrate a meaningful effect and the a (x) level was set at 0.01. Which of the following is correct?
  1. Ten per cent of the participants in the study will show an absence of clinical effect.
  2. If x = 0.01, there is a probability of 1% that the null hypothesis is wrongly rejected.
  3. It is the maximum threshold for defining clinical efficacy and therefore the clinical significance in the study.
  4. It is the minimum threshold for defining clinical efficacy and therefore the clinical significance in the study.
  5. There is a probability of a type 2 error.
34. In an audit of the length of stay in a hospital, the outcome was presented as median days. However, the analysed data showed many observations that were substantially higher than the median. Which of the following is correct?
  1. 7Mean > median
  2. Mean = median
  3. Mode = median
  4. Mode = mean
  5. Mode ≥ median
35. In conducting randomised controlled trials (RCTs), it is necessary to observe certain requirements to obtain meaningful results. The randomisation sequence is protected throughout the study until the last participant has been enrolled in the study. What is this known as?
  1. Allocation concealment
  2. Blinding
  3. Masking
  4. Matching
  5. Publication bias
36. In a meta-analysis, the results of different studies are combined to detect treatment effects. However, this could be confounded by heterogeneity in the studies. Which of the following methods is used to assess heterogeneity?
  1. Minimisation
  2. Paired Student’s t-statistics
  3. Q statistics
  4. Randomisation
  5. Stratification
37. A multicentre randomised controlled trial of a new compound thought to be an antidepressant was planned. This trial had strict inclusion criteria of hospitalised patients. Which of the following properties of this trial is most likely to be affected by strict inclusion criteria?
  1. Accuracy of the results
  2. Clinical significance of the results
  3. Desirability of the results
  4. Precision of the results
  5. Statistical significance of the results
38. A psychiatric trainee was interested in studying whether pregnant mothers who took amphetamines increased the risk of attention deficit hyperactivity disorder (ADHD) in their offspring. Which of the following statements represents the null hypothesis?
  1. Taking amphetamines during pregnancy does not increase the risk of ADHD in the child.
  2. Taking amphetamines during pregnancy increases the risk of ADHD in the child.
  3. Taking amphetamines during pregnancy decreases the risk of ADHD in the child.
  4. Taking amphetamines during pregnancy has no bearing on the risk of ADHD in the child.
  5. Taking amphetamines during pregnancy increases the risk of ADHD in the mother.
39. Which of the following is the most important methodological challenge when conducting a cohort study?
  1. Concealment of the allocation
  2. Identifying a suitable control group
  3. Identifying a suitable study group
  4. Publication bias
  5. Statistical analysis of the findings
Answer questions 40–42 by using the following information
8A 4-week RCT of aripiprazole in patients with schizophrenia was conducted; 150 patients were recruited in the aripiprazole group and 150 patients in the placebo group. Of these, 125 patients in each group completed the study. Using categorical measures of treatment response, it was reported that 70% of patients in the aripiprazole group improved whereas only 20% in the placebo group improved.
40. What is the relative risk reduction from using aripiprazole?
  1. 1.5
  2. 2.5
  3. 3.5
  4. 4.5
  5. 5.5
41. Using the results from the above study, what is the number needed to treat (NNT) for patients receiving aripiprazole compared with placebo?
  1. 2
  2. 3
  3. 4
  4. 5
  5. 6
42. What is the odds ratio of having a response using a per protocol analysis of primary outcome in the above study?
  1. 8
  2. 9.3
  3. 11.5
  4. 12.5
  5. 13.5
For each question, select one answer option
43. Which of the following statements is a limitation of the cross-sectional studies?
  1. Cross-sectional studies are prone to recollection bias.
  2. Cross-sectional studies require fewer patients.
  3. Finding a control group is very difficult.
  4. These studies are prohibitively expensive.
  5. The direction of the effect cannot be determined.
44. In the hierarchy of levels of evidence-based medicine, which of these statements is correct?
  1. An individual case–control study is on a higher level than an outcome research study.
  2. Review of case–control studies is higher than of individual cohort studies.
  3. Systematic reviews of RCTs are above systematic reviews of cohort studies.
  4. Systematic reviews of cohort studies are lower than outcome research studies
  5. Systematic reviews of cohort studies are higher than individual RCTs.
45. In the treatment of eating disorders in adolescents, family interventions were given a grade B recommendation of evidence. What type of studies does this refer to?
  1. Individual RCTs
  2. Individual case–control studies
  3. Systematic review of cohort studies
  4. Systematic review of case–control studies
  5. Systematic review of RCTs
946. Which of the following phenomena is described by Hawthorne’s effect?
  1. Individuals show improvement because they are aware of their participation in research.
  2. Smaller sample size gives false-positive results.
  3. There is a tendency for the result to be positive just by chance.
  4. The effect sizes decrease as studies are replicated.
  5. Unknown confounders modify the results.
47. Which of the following statements represents an advantage of an intention-to-treat analysis?
  1. It stops the selection biases from creeping back into the study.
  2. It allows better analysis of the data.
  3. It can make up numbers if there are a large number of dropouts.
  4. It gives more accurate results.
  5. It is ethical to include the dropouts in the final analysis.
48. Which of the following statements about replication studies is correct?
  1. Replication studies are easily published.
  2. Replication studies get ethical approval easily.
  3. Replication studies must be done using exactly the same methods as in the original study.
  4. Replication studies on a different population help to add weight to the findings.
  5. Undertaking a replication study is better than doing an original study.
49. Which of the following statements about publication bias is correct?
  1. Publishers show bias towards what interests their publications.
  2. Research with better methodology has a higher chance of being published.
  3. Research with positive results has a higher chance of being published
  4. Research with well-known experts has a higher chance of being published
  5. Researchers use contacts to get their findings published.
50. Which of the following statements about a binary variable is correct?
  1. A variable can take any value within a limited range.
  2. A variable describes a group of people.
  3. Another variable cannot be converted to binary variable.
  4. It is also known as a dichotomous variable.
  5. It is also known as an ordinal variable.
51. Which of the following statements about the Paediatric OCD Treatment Study (POTS) is correct?
  1. It was conducted in three academic centres between 1997 and 2002.
  2. It was conducted for a period of 8 weeks.
  3. It was conducted in Canada.
  4. It was conducted to compare CBT with fluoxetine in treatment of OCD.
  5. The findings suggested that fluoxetine was better than CBT.
52. Which of the following statements about the findings in the CAFÉ (Comparison of Atypicals in First Episode of psychosis) trial is correct?
  1. All-cause discontinuation was the primary outcome measure.
  2. Compared with other antipsychotics, quetiapine was more potent than others in treating the illness.
  3. A greater reduction in overall PANSS scores was seen in participants treated with quetiapine.
  4. Of the three antipsychotics compared, quetiapine was associated with the smallest elevations in fasting triglyceride and cholesterol levels
  5. Safety and tolerability were also a primary measure.
53. What does CUtLASS 1 stand for?
  1. 10Cost of Universally used Latest Antipsychotics in Schizophrenia Study 1
  2. Cost Utility of the Latest Antipsychotics in Schizophrenia Study 1
  3. Creating Universally used Latest Antipsychotics in Schizophrenia Study 1
  4. Cutting the Universal use of Latest Antipsychotics in Schizophrenia Study 1
  5. Cutting Universally used Latest Antipsychotics in Schizophrenia Study 1
54. Which of the following statements about CUtLASS 1 is correct?
  1. Clozapine and olanzapine were two of the second-generation antipsychotics.
  2. Depot preparations were excluded from the study design.
  3. More than 300 participants were assessed during the study.
  4. One of the outcome measures included scores of quality-of-life scales.
  5. Participants were from hospitals in the NHS and private sector in England
55. The objective of the research is to explore, interpret or obtain a deeper understanding of a particular clinical issue. Which of the following methods is it most appropriate to use?
  1. Case–control study
  2. Prospective cohort study
  3. Qualitative research
  4. Quantitative research
  5. Randomised controlled trial
56. Which of the following is used to compare treatments when the effect of interventions can be expressed in terms of one main variable?
  1. Cost–benefit analysis
  2. Cost-effectiveness analysis
  3. Cost-hybridisation analysis
  4. Cost-minimisation analysis
  5. Cost–utility analysis
57. Which of the following is the extent to which the results of a study are generalisable or applicable to a particular target population?
  1. Accuracy
  2. External validity
  3. Internal validity
  4. Methodological quality
  5. Precision
 
GENERAL ADULT PSYCHIATRY
58. Which of the following conditions can reliably be assessed by the Bush–Francis scale?
  1. Acute psychosis
  2. Catatonia
  3. Delirium
  4. Extrapyramidal side effects
  5. Opioid withdrawal symptoms
59. A 32-year-old man presented with a 3-month history of depressed mood, loss of energy, anhedonia, and disturbed appetite and sleep. He was also not able to concentrate. For the last 2 months, he had been experiencing auditory hallucinations asking him to take his own life. He thought that people looked at him when he went out and had become increasingly withdrawn. What is the single most likely diagnosis?
  1. 11Bipolar affective disorder, current episode severe depression with psychotic symptoms
  2. Depressive disorder with psychotic symptoms
  3. Post-schizophrenic depression
  4. Schizoaffective disorder
  5. Schizophrenia
60. In which of the following is Ganser’s syndrome more likely to be seen?
  1. Bipolar affective disorder
  2. Dementia
  3. Factitious disorder
  4. Prisoners awaiting trial
  5. Schizophrenia
61. A 21-year-old woman was known to have anorexia nervosa. Which of the following metabolic disturbances is the most likely to be found in this patient?
  1. Hyperkalaemia
  2. Impaired glucose tolerance
  3. Increased luteinising hormone (LH) levels
  4. Increased somatomedin C
  5. Reduced cortisol levels
62. A 22-year-old man with schizophrenia was admitted to an assessment ward. He was reported to be making good progress. However, his behaviour worsened whenever he returned from home leave. In the ward round, his mother complained that he was very lazy. What is the most appropriate next step to help this patient?
  1. Cognitive–behavioural therapy (CBT)
  2. Community treatment order under the Mental Health Act
  3. Counselling
  4. Family therapy
  5. Interpersonal therapy
63. Which of the following subtypes of schizophrenia is classified in DSM-IV but not ICD-10?
  1. Disorganised schizophrenia
  2. Hebephrenic schizophrenia
  3. Post-schizophrenic depression
  4. Residual schizophrenia
  5. Undifferentiated schizophrenia
64. Over the past 3 years, a 20-year-old man was becoming increasingly isolated. He spent most of his time in his room and there was some evidence of self-neglect. This was associated with a marked decline in his educational performance and inability to hold any job due to difficulty in planning and making decisions. What is the most likely diagnosis?
  1. Asperger’s syndrome
  2. Autism
  3. Catatonic personality disorder
  4. Schizoid personality disorder
  5. Simple schizophrenia
65. Which of the following is considered Valliant’s predictor of good prognosis in schizophrenia?
  1. Absence of stressful precipitating factor
  2. Absence of affective symptoms
  3. 12Chronic onset
  4. Family history of depression
  5. Schizoid traits
66. A 57-year-old man was found wandering the streets by police. He was brought to the accident and emergency department (A&E). He was considered to be in a fugue state. Which of the following is the correct statement about fugue states?
  1. Depressed mood is an extremely rare antecedent for a psychogenic fugue state.
  2. A fugue state is a syndrome consisting of a gradual loss of all autobiographical memories and knowledge of personal identity.
  3. A fugue state is usually associated with a period of wandering, for which there is a subsequent amnesic gap on recovery.
  4. Fugue states are usually not preceded by a severe precipitating stress.
  5. Fugue states are similar to transient global amnesia or transient epileptic amnesia in that the person does not know whom he or she is.
67. A 45-year-old man with a long history of alcohol dependence was diagnosed with Wernicke–Korsakoff syndrome. Which of the following statements about his condition is correct?
  1. False memories are jumbled up and retrieved appropriately, within a temporal context.
  2. It is found more commonly post mortem in people with alcohol problems than it is diagnosed in life.
  3. It is the result of nutritional depletion, namely a vitamin C deficiency.
  4. The disorder almost always has an acute onset.
  5. There is usually an anterograde memory loss.
68. A 27-year-old woman was brought to A&E. She was considered to have herpes encephalitis. Which of the following statements about herpes encephalitis is correct?
  1. Herpes encephalitis can give rise to a severe form of amnesic syndrome.
  2. Neuropathological and neuroimaging studies show that there is an extensive bilateral occipital lobe damage.
  3. Seizures are a common occurrence.
  4. The fully developed clinical picture with neck rigidity, vomiting, and motor and sensory deficits almost always occurs during the first week.
  5. The minority of cases are said to be primary infections.
69. A 34-year-old woman was considered to have intermittent uncontrollable, pathological laughter and crying spells. She was otherwise reported to be symptom free. Which of the following statements about this condition is correct?
  1. Absence of voluntary control on facial expressions
  2. Associated mood changes exaggerate response
  3. Duloxetine is an effective treatment
  4. Presence of incongruent affect
  5. Requires specific stimuli to precipitate the condition
70. A 48-year-old woman had had multiple sclerosis for some years now. She enquired about the neuropsychiatric aspects of her condition of which she should be aware. Which of the following is a neuropsychiatric manifestation of multiple sclerosis?
  1. The patient can present with subcortical dementia.
  2. Hypomania due to steroids is more likely in patients with a family history of an affective disorder.
  3. The late onset of illness is associated with a high risk of suicide.
  4. 13The lifetime prevalence of depression is 14%.
  5. Psychosis is three times more common than in the general population.
71. A 34-year-old woman presented with tiredness for more than a year. She had been off sick for more than 6 months and was unable to carry on with her routine activities. She had poor concentration, poor memory, headaches, sore throat and joint pains. What is the most likely diagnosis?
  1. Chronic fatigue syndrome
  2. Depression
  3. Hypothyroidism
  4. Factitious disorder
  5. Somatisation disorder
72. Schizotaxia is:
  1. Ataxia in schizophrenia
  2. Familial schizotypal disorder
  3. Genetic constitution increasing vulnerability to schizophrenia
  4. Neuropsychiatric manifestations in schizophrenia
  5. Slow cognitive decline in schizophrenia
73. Which of the following is a correct match of medical condition associated with a secondary sleep problem?
  1. Dementia–nocturnal myoclonus
  2. Epilepsy–parasomnias
  3. Huntington’s disease–hypersomnia
  4. Kleine–Levin syndrome–difficulty initiating sleep
  5. Parkinsonism–sun downing
74. In which of the following is the ‘match box’ or ‘pill bottle’ sign seen?
  1. Delusions involving the skin
  2. Delusions of body odour
  3. Delusions of sexually transmitted infection
  4. Delusions of ugliness
  5. Pyromania
75. A 37-year-old man with a persistent delusional disorder attended your outpatient clinic. You discussed his treatment options and prognosis with him. Which of the following is a correct statement about this?
  1. A delusional system ameliorates after just 2–3 months.
  2. The maintenance antipsychotic dose is usually very high.
  3. Once treated successfully, if the patient stops medication he is likely to remain stable for around 6–12 months before he may relapse.
  4. Overall, the best-attested treatment result refers to somatic subtype.
  5. Recovery is often very slow.
76. In which of the following conditions is an accentuation of a normal EEG seen?
  1. Alzheimer’s disease
  2. Creutzfeldt–Jakob disease
  3. Huntington’s disease
  4. Presenile Alzheimer’s disease
  5. Pseudo-dementia
1477. A 40-year-old man was recently diagnosed with Wilson’s disease. He enquired about the possible psychiatric manifestations of his condition of which he should be aware. Which of the following is correct about Wilson’s disease?
  1. Cognitive impairment
  2. Depression
  3. Disorientation
  4. Personality change and incongruous behaviour
  5. Psychotic behaviour
78. A 62-year-old woman with systemic lupus erythematosus (SLE) was referred to your outpatient clinic. She enquired about the possible neuropsychiatric manifestations of her condition. Which of the following statements about SLE is correct?
  1. Headaches are most uncommon.
  2. They are characteristic features of SLE.
  3. They are usually long-term manifestations.
  4. They show a tendency to appear in the later stages of the disease.
  5. They usually reoccur when there are no other systemic features.
79. A 42-year-old woman with hypothyroidism presented in your outpatient clinic. She would like to know about the neuropsychiatric manifestations of her condition. Which of the following statements about hypothyroidism is correct?
  1. Auditory hallucinations are less common than other types of perceptual abnormalities.
  2. Depression with psychotic symptoms responds readily to treatment with psychotropic medications.
  3. Memory is intact to a large extent in the early stages.
  4. Mood is depressed rather than manic.
  5. Paranoid features figure prominently in psychosis.
80. A 52-year-old woman with Cushing’s syndrome presented in your outpatient clinic. She would like to discuss the psychiatric manifestations of her condition. Which of the following statements about Cushing’s syndrome is correct?
  1. Depression is the most common psychiatric presentation in Cushing’s syndrome due to pituitary involvement.
  2. Depression is the most common psychiatric presentation in Cushing’s syndrome due to adrenal carcinoma or adenoma.
  3. Psychiatric disturbances are found in only 20–25% of cases.
  4. Psychosis often presents without any flavour of paranoia but mainly with auditory hallucinations.
  5. Psychotic depression is the most common manifestation.
81. A 39-year-old man presented in the initial stage of Huntington’s disease in your outpatient clinic. He wanted to know about the psychiatric manifestations and likely prognosis of his condition. Which of the following statements is correct?
  1. Dementia precedes involuntary movements.
  2. It follows a more severe course when the onset is early.
  3. Psychiatric changes usually occur once the diagnosis has been fully established.
  4. The ratio of incidence is male:female = 6:1.
  5. The abnormal movements continue to a lesser extent during sleep.
82. Which of the following statements about parasuicide is correct?
  1. It is usually an impulsive act.
  2. Life events are common in the 6 months before an act of parasuicide.
  3. 15Most cases of parasuicide are associated with malingering.
  4. Parasuicide was first defined by Melanie Klein.
  5. A person injures him- or herself by taking a substance in a quantity that is less than the therapeutic dose.
83. A 29-year-old woman with body dysmorphic disorder (BDD) attended your clinic for a review. She would like to learn more about her condition. Which of the following statements about BDD is correct?
  1. Anorexia nervosa does not fulfil the diagnostic criteria for BDD.
  2. In psychodynamic terms, BDD has not been considered to represent an unconscious displacement on to body parts of sexual or emotional conflicts, or of general feelings of inferiority, poor self-image or guilt.
  3. It is a delusional belief of slight or perceived defect of one’s body.
  4. Studies of college students have suggested that a quarter meet DSM-IV criteria for BDD.
  5. The most common age of onset is the third decade of life.
84. A 35-year-old woman attended your clinic with multiple psychosomatic symptoms. Which of the following statements is least likely to describe the medically unexplained physical symptoms?
  1. About a quarter of new consultations in secondary care will be for medically unexplained symptoms.
  2. In any week, 60–80% of healthy people experience bodily symptoms.
  3. Most patients in clinical practice with unexplained physical symptoms have either an undifferentiated somatoform disorder or a primary psychiatric disorder.
  4. Somatic symptoms of pain in the face are considered psychodynamically equivalent to an emotional ‘slap in the face’.
  5. Symptoms are transient but a third persist and cause distress and disability.
85. A 21-year-old man was diagnosed with schizophrenia. He was worried about his twin brother becoming mentally ill. What is the likely risk of developing schizophrenia in the identical twin of a person with schizophrenia?
  1. 0–10%
  2. 10–20%
  3. 20–30%
  4. 30–40%
  5. 40–65%
86. A 24-year-old woman was diagnosed with pseudo-seizures. Which of the following statements about this condition is correct?
  1. Of patients 40–50% have coexisting epilepsy.
  2. Clinical observation can differentiate easily between pseudo-seizures and seizures.
  3. They are a post-ictal impaired papillary reflex.
  4. They are a post-ictal decrease in prolactin concentration.
  5. Urinary incontinence can occur in this condition.
87. Which of the following is a characteristic feature of the Gastaut–Geschwind syndrome?
  1. Hyperreligiosity
  2. Hypersexuality
  3. Hypographia
  4. Mainly seen in absent seizure
  5. Tangential
16
 
OLD AGE PSYCHIATRY
88. A 76-year-old woman presented in your outpatient clinic with paranoid symptoms and general decline in her personal hygiene. A diagnosis of late-onset schizophrenia was considered as most likely in the absence of any organic brain pathology. Which of the following is correct?
  1. Better response to antipsychotics
  2. Increased likelihood of affective blunting
  3. Increased likelihood of formal thought disorder
  4. Less risk of developing tardive dyskinesia
  5. More negative than positive symptoms
89. A 68-year-old man had had a mental disorder for a long time. He wished to discuss the long-term implications of his condition because he had read that he carried an increased risk for vascular disease in later life. Which of the following disorders is he most likely to have at present?
  1. Bipolar affective disorder
  2. Generalised anxiety disorder
  3. Obsessive–compulsive disorder
  4. Recurrent depression
  5. Schizophrenia
90. A 70-year-old woman had treatment-resistant depression. She consented to a course of electroconvulsive therapy (ECT). Which of the following factors leads to an increase in the seizure threshold?
  1. Being a female patient
  2. Co-administration of antidepressants
  3. Co-administration of antipsychotics
  4. Increasing age
  5. Past history of ECT 5 years ago
 
CHILD AND ADOLESCENT PSYCHIATRY
91. A 10-year-old boy presented in your clinic with his mother and stepfather. His parents were worried that he was disruptive, defiant and challenging most of the time at home. They wanted to know why he was like this and different from his two siblings. Which of the following is the most relevant risk factor in the development of conduct disorder in children?
  1. Drinking during pregnancy
  2. Parents with a history of conduct disorder
  3. Parents with a forensic history
  4. Siblings with a forensic history
  5. Urban areas
92. A 13-year-old girl presented with her mother in your clinic with panic attacks and difficulties with breathing. She wanted to know what was wrong with her and also the prevalence of mental disorders in children and adolescents. Which of the following statements is correct?
  1. Community prevalence is around 10–15%.
  2. Figures of overall prevalence are similar across cultures around the world.
  3. Prevalence of moderate-to-severe illnesses is around 6–8%.
  4. Proportion of girls with psychiatric diagnoses relative to boys reduces with age.
  5. Prevalence of girls with psychiatric disorders is now higher than for in boys.
1793. A 16- year-old girl was treated for depression by her GP. She presented in your clinic to discuss the long-term implications of her depression. Which of the following statements about depression in children and adolescents is correct?
  1. Childhood depression is associated with higher rates of depression in adult life.
  2. Depression is less common in boys than in girls up to adolescence.
  3. Point prevalence of depression in adolescents is 7–8%.
  4. Point prevalence of depression in children is 5%.
  5. There is a high rate of recovery but a low rate of relapse in children.
94. A 9-year-old boy attended your clinic with his mother. You would like to discuss parent–child interaction therapy (PCIT), which is a parent training programme, with the mother. Which of the following is a correct statement about PCIT?
  1. The aim of the therapy is to teach a child how to behave appropriately.
  2. It was developed by Fonagy at the University of Florida.
  3. It consists of five phases.
  4. Teaching begins at the clinic and then is gradually taken home.
  5. The therapist takes turns with the parent in playing with the child.
95. A 10-year-old boy with a diagnosis of autism presented with significant irritability, aggression and self-injury. At assessment, no specific underlying cause could be found. After a trial of behaviour management was unsuccessful, it was decided to try a medication. Which of the following medications is considered effective in such a situation?
  1. An antihistamine
  2. Buspirone
  3. Fluoxetine
  4. Methylphenidate
  5. Risperidone
96. A 15-year-old boy was treated for depression by his GP. Unfortunately, his condition got worse. He now presents in your clinic. Which of the following is a first-line treatment?
  1. Dynamic psychotherapy
  2. Family therapy
  3. Fluoxetine
  4. Group therapy
  5. Sertraline
97. A 14-year-old girl who took an overdose of paracetamol and bleach for no apparent reason was brought to the accident and emergency department. She wanted to know why she had tried to harm herself. She also wanted to know how many other children and adolescents self-harm every year. Approximately what proportion of children and adolescents with self-harm present themselves to hospitals in England?
  1. 13%
  2. 26%
  3. 45%
  4. 62%
  5. 89%
98. A 7-year-old persistently refused to go to school. What is the most common underlying disorder?
  1. Autism
  2. Depression
  3. Panic disorder
  4. Separation anxiety
  5. Specific phobia
1899. A 14-year-old severely depressed boy was treated with a combination of CBT and fluoxetine. He was feeling better but could not tolerate fluoxetine’s side effects. Which of the following would be the est option to consider as the second line?
  1. Citalopram
  2. Escitalopram
  3. Mirtazapine
  4. Paroxetine
  5. Venlafaxine
100. A 6-year-old boy with autism attended your clinic for a review. His parents would like to know the long-term outcome of their son’s condition. Which of the following statements about the prognosis of autism is correct?
  1. Autistic individuals are at an increased risk of developing schizophrenia in adult life.
  2. Autistic children with reasonable speech and normal IQ have good social outcomes in >70% of cases.
  3. Autistic aloofness does not improve with time.
  4. A third of autistic children who do not have useful speech by age 5 will go on to develop it later in life.
  5. About 10% of autistic individuals go through a phase in adolescence when they lose language skills.
101. A 10-year-old girl was diagnosed with a psychiatric condition. Her parents blamed themselves for her condition. You tried to offer some information to them. Which of the following conditions is thought to have a proportionally greater influence of a shared environment than shared genes?
  1. Autism
  2. Conduct disorder
  3. Depression
  4. Hyperkinetic disorder
  5. Schizophrenia
102. A 14-year-old boy with a conduct disorder was brought to your clinic. You discussed his care plan with his parents. Which of the following is considered the best-established approach to manage conduct disorder?
  1. Cognitive–behavioural therapy
  2. Group psychotherapy
  3. Interpersonal psychotherapy
  4. Medication
  5. Parent management training
 
LEARNING DISABILITY
103. A 39-year-old primipara woman attended your clinic for genetic counselling because she has a family history of an inherited genetic condition. Which of the following genetic conditions is most unlikely to be inherited?
  1. Down’s syndrome
  2. Edwards’ syndrome
  3. Fragile X syndrome
  4. Klinefelter’s syndrome
  5. Turner’s syndrome
19104. A 30-year-old man with Down’s syndrome attended your clinic. He was in a relationship with a 28-year-old woman who also had Down’s syndrome. He wanted to know about the chances of having a child. Which of the following statements about fertility in people with Down’s syndrome is correct?
  1. Men with Down’s syndrome due to non-disjunction have a high incidence of infertility.
  2. Men with mosaic Down’s syndrome have a high incidence of infertility.
  3. Women with Down’s syndrome due to non-disjunction have a high incidence of infertility.
  4. Women with Down’s syndrome due to robertsonian translocation have a high incidence of infertility.
  5. Women with mosaic Down’s syndrome have a high incidence of infertility.
105. A 30-year-old man with schizophrenia and learning disability attended your clinic with his support worker for a review. The support worker wanted to know more about the relationship between mental disorders and learning disability. What is the prevalence of psychiatric disorders (excluding behavioural problems) in the learning-disabled population?
  1. 5–20%
  2. 20–40%
  3. 40–50%
  4. 50–70%
  5. 70–80%
106. A 24-year-old man with a severe learning disability was brought by his support worker to your clinic for a review. The support worker wanted to know more about severe learning disability and its implications for the individual and society. Which of the following statements about the epidemiology of severe intellectual impairment is correct?
  1. Age-specific prevalence varies over time.
  2. Point prevalence is similar between similar birth cohorts in different communities within the UK.
  3. Studies have failed to show a social class gradient consistent with the known social distribution of morbidity and mortality.
  4. There is usually the same proportion of males and females at all ages.
  5. Very different patterns of temporal variation have occurred throughout the developed world.
107. Which of the following is measured by the Life Experiences Checklist?
  1. Disability
  2. Handicap
  3. Impairment
  4. Learning disability
  5. Mental illness
108. Which of the following statements about offending in the learning-disabled (LD) population is correct?
  1. Evidence exists for increased rates of violence in this population.
  2. Learning disability together with obsessive personality features carries a high risk for offending.
  3. Offending is more likely in the severe LD group than the mild-to-moderate group.
  4. Offences are broadly similar to those of offenders without learning disability.
  5. Property offences are committed with excessive forethought.
20109. A 7-year-old girl was diagnosed with mild learning disability. She attended your clinic for a review. Her mother had heard about subcultural learning disability. Which of the following sstatements bout subcultural learning disability is correct?
  1. Family members are more likely to have borderline or mild intellectual disability.
  2. It has no relationship with the socioeconomic status of the family in which the child was born.
  3. There are dysmorphic features in children with subcultural learning disability.
  4. There will be impaired adaptive functioning.
  5. There will be a severe degree of learning disability in some family members.
110. An 8-year-old boy studying in year 3 was referred to the learning disability services. Over the past year, he had developed partial blindness, myoclonic jerks and intellectual deterioration. What is the most likely diagnosis?
  1. Batten’s disease
  2. Down’s syndrome
  3. Parry’s disease
  4. Rett’s syndrome
  5. Williams’ syndrome
111. A 10-year-old boy was diagnosed with dyslexia. He attended your clinic for a review. His father who accompanied him would like to know what deficits the boy is likely to encounter in the future. Which of the following is the primary skill deficit in patients with dyslexia?
  1. Higher linguistics
  2. Impaired rapid automatic naming
  3. Phonological working memory
  4. Single word reading
  5. Syntactic and semantic processing skills
 
FORENSIC PSYCHIATRY
112. You were asked to assess a prisoner who had been charged with violent offences against members of the public. To complete a comprehensive risk assessment, you decided to use an actuarial tool for violence risk assessment. Which of the following is the most appropriate tool for this purpose?
  1. ESR-20
  2. HCR-20
  3. PCL-R (Hare)
  4. RSVP
  5. SVR-20
113. A 24-year-old man was charged with a criminal offence. However, he pleaded not guilty to the charge. In British law, all offences must be proven to confirm that the accused physically did the act. Which of the following is the most appropriate legal term?
  1. Actus rea
  2. Actus reus
  3. Arcus rea
  4. Mens actus
  5. Mens rea
21114. According to current research evidence, what is the percentage of convicted prisoners with psychosis?
  1. 4%
  2. 6%
  3. 8%
  4. 10%
  5. 15%
115. According to current research evidence, what is the estimated percentage of homicides that are followed by suicide of the perpetrator (homicide–suicide)?
  1. 0.5%
  2. 1%
  3. 2%
  4. 5%
  5. 10%
116. A 26-year-old man was charged with a violent offence against his girlfriend. However, he pleaded not guilty to the charge because he claimed that it occurred when he was sleepwalking. For his explanation to be plausible, during which stage of sleep does sleepwalking occur?
  1. Rapid eye movement (REM) sleep
  2. Stage 2
  3. Stages 1–2
  4. Stages 2–3
  5. Stages 3–4
117. A 35-year-old man with Asperger’s syndrome was charged with a public disorder offence. His solicitors would like to know why he might have committed the alleged offence. They also wanted to know which category of offences individuals with Asperger’s syndrome are more likely to commit compared with individuals without it.
  1. Acquisitive
  2. Arson
  3. Drug offences
  4. Fraud
  5. Sexual offences
118. Which of the following statements about Pritchard’s criteria to assess fitness to plead in the UK is correct?
  1. It applies criteria at the time of the alleged offence.
  2. Being unfit to plead can result in a trial of facts.
  3. It includes an assessment for fitness to appear in court.
  4. People who are mute are automatically found unfit to plead.
  5. It requires an understanding of previous trials of a similar nature.
119. Which of the following statements about insane automatism in the UK is correct?
  1. It can be caused by an insulinoma.
  2. A defendant has partial control over his actions.
  3. It includes confusional states following anaesthesia.
  4. Intent to harm does not need to be excluded.
  5. The McNaughton rules do not apply.
22
 
SUBSTANCE MISUSE/ADDICTIONS
120. A 43-year-old man presented with an alcohol problem. He wanted to know more about his condition including the prognosis. You discussed Jellinek’s classification of alcohol problems and considered that he had b alcoholism. Which of the following is a characteristic feature of his condition?
  1. Absence of withdrawal symptoms
  2. Dipsomania
  3. Physical dependence and loss of control
  4. Polyneuropathy and cirrhosis of the liver
  5. Psychological dependence, drinking to relieve emotional pain
121. According to ICD-10, which of the following statements is correct with regard to criteria for a dependence syndrome?
  1. This involves persisting with a substance with or without evidence of harm.
  2. A physiological withdrawal state should be evident only when substance use has ceased.
  3. A strong desire to take the substance is present every day.
  4. Three or more manifestations must be present for more than a month.
  5. Tolerance refers to diminished effects with reduced intake of substance.
122. According to ICD-10, which of the following is a character code that may be used to further specify dependence syndrome?
  1. Currently remitting
  2. Currently dependent after partial remission
  3. Currently dependent but receiving treatment
  4. Late remission
  5. Partial remission
123. A 35-year-old man presented with mild alcohol dependence. Which of the following would be the most appropriate psychological intervention for him?
  1. Cognitive–behavioural therapy
  2. Family therapy
  3. Group therapy
  4. Interpersonal therapy
  5. Psychodynamic psychotherapy
124. Which of the following is the proposed mechanism of action of acamprosate in alcohol dependence?
  1. GABA agonism
  2. Glutamatergic agonism
  3. NMDA agonist
  4. Noradrenergic antagonism
  5. Serotonin activation
 
PSYCHOTHERAPY
125. A 23-year-old woman presented with bulimia nervosa in the eating disorders clinic. She would like to know about various treatment options for her condition. Which of the following treatment options for bulimia nervosa has the best evidence base?
  1. Dialectical behavioural therapy
  2. Family therapy
  3. 23Self-help
  4. Serotonin reuptake inhibitors
  5. Specialised cognitive–behavioural therapy
126. A 37-year-old man with Capgras’ syndrome attended your clinic for a review. His care coordinator who would like to learn more about this condition accompanied him. Which of the following statements about Capgras’ syndrome is correct?
  1. It occurs in organic pathological states.
  2. It involves strangers.
  3. It is common among individuals with schizophrenia.
  4. The abnormality is hallucinatory.
  5. There is a perceived outward change in a person’s appearance.
127. You assessed a 45-year-old man with a chronic alcohol problem and morbid jealousy. His partner, who is the mother of his three children, accompanied him. She would like to learn about his condition. Which of the following statements about morbid jealousy is correct?
  1. It is a common motivation for homicide.
  2. It is also known as Hamlet’s syndrome.
  3. It is almost always delusional in nature.
  4. It occurs in women more than men.
  5. Violence is more often vented on the supposed rival than on the partner.
128. A 25-year-old man complained of seeing roof tiles as a brilliant flaming red. He did not complain of any other psychiatric symptoms. Which of the following terms describes this phenomenon?
  1. Dysmegalopsia
  2. Hyperaesthesia
  3. Hallucination
  4. Hyperschemazia
  5. Illusion
129. A 32-year-old man attended your outpatient clinic with his 6-month pregnant wife. He informed you that he could feel his unborn baby’s movements in his abdomen. What is this phenomenon known as?
  1. Autoscopy
  2. Cotard’s syndrome
  3. Couvade’s syndrome
  4. Irritable bowel syndrome
  5. Pseudocyesis
130. A depressed 30-year-old woman was assessed and it was considered that she needed to work on her unresolved grief. Her problems were linked to a number of stressors, characteristic patterns of relationships and self-limiting themes that were complex and interactive. Which of the following therapies would be best suited to her needs?
  1. Cognitive–analytic therapy
  2. Cognitive–behavioural therapy
  3. Group therapy
  4. Long-term individual therapy
  5. Short-term dynamic therapy
24
 
Questions: EMIs
 
RESEARCH METHODS, STATISTICS, CRITICAL REVIEW AND EVIDENCE-BASED PRACTICE
 
Theme: Comparison of relationships between different variables
Options for questions: 131–135
  1. Correlation coefficient
  2. Kendall’s correlational coefficient
  3. Logistic regression
  4. Multiple analysis of co-variance
  5. Multiple analysis of variance
  6. Multiple linear regression
  7. Pearson’s correlational coefficient
  8. Proportional Cox’s regression
  9. Simple linear regression
  10. Spearman’s rank correlation coefficient
For each of the following, select the single most appropriate application. Each option may be used once, more than once or not at all.
131. It indicates degree to which two measures are related although it does not explain how.
132. It is a correlational coefficient for two categorical or non-parametrically (non-normally) distributed variables.
133. It is used when the dependent variable is of a dichotomous or binary type (e.g. yes or no), whereas the independent variable can be of any type.
134. It is used with multiple dependent and independent variables.
135. It is used with multiple dependent variables.
 
Theme: Bias and confounders in research studies
Options for questions: 136–140
  1. Berksonian bias
  2. Blinding
  3. Matching
  4. Randomisation
  5. Recall bias
  6. Restriction
  7. Stratification
  8. Systemic sampling
For each of the following situations, select the single most appropriate bias or confounder. Each option may be used once, more than once or not at all.
136. When researchers were evaluating patient’s responses, they were kept unaware whether the patients were from the control arm or the experimental arm.
137. A structured interview was used with standardised criteria for diseases and exposures. The information was obtained from more than one source.
138. In a study of schizophrenia, the researchers recruited equal number of cannabis smokers and non-smokers.
139. In a different study of schizophrenia, the researchers excluded all cannabis users from the study and explored variables.
140. In a population-based study of incidence of schizophrenia in the UK, the researchers looked at groups from different income brackets.
25
 
GENERAL ADULT PSYCHIATRY
 
Theme: Psychotic phenomenology in mental disorders
Options for questions: 141–144
  1. Auditory hallucination
  2. Charles Bonnet syndrome
  3. Concrete awareness
  4. Delirium tremens
  5. Delusional percept
  6. Depression without psychosis
  7. Formal thought disorder
  8. Haptic hallucinations
  9. Mania without psychotic features
  10. Synaesthesia
  11. Temporal lobe epilepsy (TLE)
  12. Visceral hallucinations
For each of the following cases, select the single most appropriate description. Each option may be used once, more than once or not at all.
141. A 22-year-old man claimed that he felt, saw, tasted and smelt the music from a trumpet.
142. A 48-year-old man with deteriorating diabetic retinopathy had recently been horrified by visual hallucinations of snakes and green serpents trying to bite him.
143. A 35-year-old Indian man with a history of febrile convulsions was struck on his head by a cricket ball recently. Since then, he has complained of visual hallucinations of seeing his old time friend often talking to him in their local language.
144. A 41-year-old man presented to A&E as experiencing illusions that were frequently associated with hallucinations. They changed so rapidly that it was difficult for him to describe them and he was frightened most of the time.
 
RESEARCH METHODS, STATISTICS, CRITICAL REVIEW AND EVIDENCE-BASED PRACTICE
 
Theme: Research methodology and clinical studies
Options for questions: 145–150
  1. CAFÉ
  2. CATIE
  3. CUtLASS 1
  4. DEPRES
  5. ESEMED
  6. IPSS
  7. MTA
  8. POTS
  9. SOHO
  10. STAR*D
  11. TEOSS
  12. TORDIA
For each of the following, select the single most appropriate research study. Each option may be used once, more than once or not at all.
145. First-generation antipsychotic used was perphenazine
146. An observational study conducted in Europe, funded by Eli Lilly, on antipsychotics for schizophrenia
147. Study on depression in Europe funded by SmithKline Beecham Pharmaceuticals
148. Study conducted in six countries across Europe, excluding the UK. Used CIDI to generate questionnaires
26149. Study comparing sertraline, CBT and a combination for treatment of OCD in children
150. Study comparing first-generation antipsychotics (FGAs) against second-generation antipsychotics (SGAs) in schizophrenia
 
Theme: Epidemiological concepts
Options for questions: 151–153
  1. Cumulative incidence
  2. Life-time prevalence
  3. Morbidity rate
  4. Point prevalence
  5. Period prevalence
  6. Standardised mortality rate
  7. Standardised mortality ratio
For each of the following cases, select the single most appropriate application. Each option may be used once, more than once or not at all.
151. In your catchment area, you decided to screen all primary schoolchildren for ADHD and found that about 5% of school children had ADHD symptoms.
152. The proportion of participants who had developed a condition within a specified time period.
153. Mortality rate that is adjusted to compensate for age, sex, etc.
 
Theme: Randomisation methods in research studies
Options for questions: 154–157
  1. Minimisation
  2. Play the winner
  3. Simple randomisation
  4. Stratified randomisation
  5. Randomised consent method
  6. Randomisation permutated blocks
For each of the following cases, select the single most appropriate randomisation method. Each option may be used once, more than once or not at all.
154. In this method, the first participant is allocated by a simple randomisation procedure and then after every subsequent allocation it is based on the success or failure of the immediate predecessor participant.
155. This technique is useful for studies of small size and long duration in which it can ensure that roughly comparable numbers of participants are allocated to study groups at any point in the process.
156. In this method, an eligible population is divided according to a minimum number of important prognostic factors.
157. This method is used in some clinical trials to lessen the effect of some patients refusing to participate.
 
Theme: Design features of clinical research studies
Options for questions: 158–160
  1. Control participants receive a placebo
  2. Each group receives a different treatment with both groups being entered at the same time
  3. Each participant received both the intervention and the treatment
  4. Participants are assessed before and after an intervention
  5. 27Results are analysed by comparing groups
  6. Results are analysed in terms of differences between participant pairs
  7. Separated by a period of no treatment
  8. This permits an investigation of the effects of more than one independent variable on a given outcome
For each of the following studies, select TWO most appropriate features. Each option may be used once, more than once or not at all.
158. Cross-over trial
159. Parallel-group comparison
160. SPaired comparison
 
Theme: Randomised control trials
Options for questions: 161–163
  1. Assumes a poor outcome for drop-outs
  2. ‘Brought forward’ data are incorporated into the overall analysis of whichever group they originally belonged to
  3. Data should be analysed as a unit of randomisation
  4. Data on all patients entering a trial should be analysed with respect to the groups to which they were originally randomised, regardless of whether or not they received treatment
  5. Differences in the drop-out rates and the timing of these drop-outs influence the estimation of treatment
  6. Interventions are directed at groups rather than individual participants
  7. This may not be appropriate in trials assessing the physiological effects of a drug as opposed to efficacy
For each of the following descriptions, select TWO most appropriate options. Each option may be used once, more than once or not at all.
161. Last data carried forward
162. Intention-to-treat analysis
163. Cluster trials
 
GENERAL ADULT PSYCHIATRY
 
Theme: Costs involved in healthcare
Options for questions: 164–166
  1. Avoiding hospital admission
  2. Investigations
  3. Pain and suffering
  4. Prevention of expensive-to-treat illness
  5. Social stigma
  6. Staff salaries
  7. Value of ‘unpaid work’
  8. Work days lost
For each of the following descriptions, select the TWO most appropriate cost options. Each option may be used once, more than once or not at all.
164. Direct costs
165. Intangible costs
166. Indirect costs
 
Theme: Medical databases
Options for questions: 167–169
  1. 28Allied and Complementary Medicine Database (AMED)
  2. Cumulative Index to Nursing and Allied Health Literature (CINAHL)
  3. Cochrane Library
  4. EMBASE
  5. Google Scholar
  6. Health STAR
  7. MEDLINE
  8. PsycINFO
  9. Scopus
For each of the following descriptions, select the single most appropriate database. Each option may be used once, more than once or not at all.
167. It covers health services, hospital administration and health technology assessment.
168. This is a nursing and allied health database.
169. The database of Excerpta Medica focuses on drugs and pharmacology, clinical medicine and other biomedical specialties.
 
Theme: Architecture of clinical research
Options for questions 170–172
  1. Causation can be inferred
  2. Examples include controlled clinical trial and economic analyses, systematic reviews and their meta-analyses
  3. Examples include case–control and cohort studies
  4. Examples include case reports and series, audits, cross-sectional surveys and qualitative study
  5. Generally compare two groups
  6. Generally conducted without a control group
  7. Something is given or done in the experimental group but not in the control group
  8. Suitable for hypothesis generation
  9. Suitable for hypothesis testing
For each of the following studies, select THREE most appropriate descriptions. Each option may be used once, more than once or not at all.
170. Descriptive studies
171. Analytical studies
172. Experimental studies
 
RESEARCH METHODS, STATISTICS, CRITICAL REVIEW AND EVIDENCE-BASED PRACTICE
 
Theme: Sources of bias in clinical drug trials
Options for questions: 173–176
  1. Double blinding
  2. Information bias
  3. Observer bias
  4. Recall bias
  5. Selection bias
  6. Single blinding
  7. Triple blinding
For each of the following descriptions, select the single most appropriate source of bias. Each option may be used once, more than once or not at all.
29173. It reduces the potential bias of a patient’s placebo response.
174. It reduces the doctor’s sometimes overzealous desire to find a good new treatment.
175. The analysis of outcomes is conducted by an independent researcher.
176. This is prone to non-blind outcome assessment.
 
Theme: Measures of disease frequency
Options for questions: 177–180
  1. Incidence density
  2. Incidence risk
  3. Lifetime risk
  4. Mortality ratio
  5. Period prevalence
  6. Point prevalence
  7. Standardised mortality ratio
For each of the following cases, select the single most appropriate option. Each option may be used once, more than once or not at all.
177. The number of new cases of the disease over a period of time out of the total population at risk.
178. The number of new cases out of the person-time of observation.
179. The number of individuals with the disease in the population over a period of time.
180. The ratio of observed to expected deaths.
 
Theme: Sources of error in tests
Options for questions: 181–183
  1. Bias towards the centre
  2. Extreme responding
  3. Halo effect
  4. Hawthorne’s effect
  5. Response set
  6. Social acceptability
  7. Recollection bias
For each of the following cases, select the single most appropriate source of error. Each option may be used once, more than once or not at all.
181. The participants will always either agree or disagree with the questions and hence the true opinion might not be revealed.
182. The participant chooses answers that might not offend others or the ones that examiners want to know.
183. The participants’ responses are altered by the mere presence of the examiner.
 
GENERAL ADULT PSYCHIATRY
 
Theme: Frontal lobe brain tumours and respective symptomatology
Options for questions: 184–186
  1. Anterior cingulate tumours
  2. Bilateral frontal lobe tumours
  3. Dorsolateral prefrontal convexity tumours
  4. 30Left frontal lobe tumours
  5. Orbitofrontal tumours
  6. Tumour of falx
For each of the following cases, select the single most appropriate diagnosis. Each option may be used once, more than once or not at all.
184. Impulsivity with behavioural disinhibition
185. Apathy and lack of spontaneity
186. Akinetic mutism
 
Theme: Contributions to psychiatry
Options for questions: 187–190
  1. Benedict Morel
  2. Eugene Bleuler
  3. Emil Kraepelin
  4. Jacob Kasanin
  5. John Cade
  6. Karl Ludwig Kahlbaum
  7. Phillepe Pinel
  8. RD Laing
For each of the following cases, select the single most appropriate person. Each option may be used once, more than once or not at all.
187. Associated with the anti-psychiatry movement
188. Coined the term démence precoce
189. Described catatonia for the first time
190. Discovered the effects of lithium as a mood stabiliser
 
Theme: Cognitive distortions
Options for questions: 191–193
  1. Arbitrary inference
  2. Catastrophising
  3. Dichotomous thinking
  4. Disqualifying the positive
  5. Magical thinking
  6. Minimisation
  7. Overgeneralisation
  8. Personalisation
  9. Selective abstraction
For each of the following descriptions, select the single most appropriate cognitive distortion. Each option may be used once, more than once or not at all.
191. A 30-year-old, hard-working and sincere employee greeted you and complimented you on your achievements. You said that he was doing it to get a promotion in your company.
192. A 16-year-old girl decided not to ask a girl in her class to join her for shopping because she knew that she would say no.
193. A 14-year-old boy played football for his school. He scored a goal and his team won the match. He told his mother that during the game he tripped and almost let in a goal for the other side. He said that he was rubbish at football and did not wish to play again.
31
 
CHILD AND ADOLESCENT PSYCHIATRY
 
Theme: First-line medication for disorders in children and adolescents
Options for questions: 194–196
  1. Acute mania
  2. Conduct disorder
  3. Mild ADHD without co-morbidity
  4. Mild depression
  5. Moderate depression
  6. Schizophrenia
  7. Severe ADHD without co-morbidity
  8. Gilles de la Tourette’s syndrome
  9. Young people with borderline personality disorder
For each of the following drugs, select the single most appropriate diagnosis for which it is indicated. Each option may be used once, more than once or not at all.
194. Fluoxetine
195. Methylphenidate
196. Olanzapine
 
LEARNING DISABILITY
 
Theme: Behavioural symptoms associated with learning disability
Options for questions: 197–200
  1. Angelman’s syndrome
  2. Cornelia de Lange’s syndrome
  3. Down’s syndrome
  4. Fragile X syndrome
  5. Lesch–Nyhan syndrome
  6. Prader–Willi syndrome
  7. Rett’s syndrome
  8. Tuberous sclerosis
  9. Velocardiofacial syndrome
  10. Williams’ syndrome
For each of the following cases, select the single most appropriate diagnosis. Each option may be used once, more than once or not at all.
197. Verbal and physical aggression; self-injurious behaviour such as biting lips, inside of the mouth and fingers, thumping of the ears and face, hitting the head against objects; severe learning disability; hypertonia, ataxia and involuntary movements.
198. Abnormalities in speech; cognitive impairment with relatively intact visuospatial skills, reduced short-term memory and greater loss from memory over time; sleep abnormalities; good tempered and cheerful; severe behavioural problems associated with hyperphagia, self-injurious behaviour particularly in the form of incessant spot picking and temper tantrums.
199. Severe or profound learning disability; lack of speech; ataxia; inappropriate bouts of laughter; tongue thrusting; hand flapping and mouthing behaviour.
200. Episodes of low mood; episodes of anxiety with hyperventilation, screaming, self-injury, a frightened expression and general distress precipitated by sudden noises, certain music, strange people or places, changes in routine and excessive environmental activity; self-injurious behaviour associated with abnormal hand movements including biting and chewing of fingers and hands; sleep problems including laughing at night and autistic features.
32
 
Answers: MCQs
 
1. B There should be at least one trained nurse in the recovery area
There should be at least one trained nurse in the treatment area. The ECT team has to be same every week in the clinic. It is the ECT psychiatrist who develops protocol for ECT prescription. There should be at least one person competent in cardiopulmonary resuscitation for every unconscious patient.
 
2. B An ‘ECT rights about consent to treatment’ leaflet is provided to detained patients
Consent should be obtained by the referring psychiatrist and checked by the ECT psychiatrist before administering any treatment. ECT information must be provided both verbally and in written form. Explanation on the adverse effects of ECT on cognition must be provided. Capacity assessment must be documented for all patients, including voluntary ('informal') patients.
 
3. B Increased appetite
The clinical predictors of a response to phototherapy include increased sleep, winter weight gain, carbohydrate craving, afternoon slump in energy and complete remission in the summer. Lack of bright light in winter months is considered a major cause of seasonal affective disorder, and is also known as ’winter blues‘. Light entering our eyes stimulates the daily mood rhythm in the brain by hormone production. Although light therapy is a popular treatment, there is no strong evidence to support its long-term benefit.
 
4. E The intensity of rTMS is usually set as a percentage of the patient’s motor threshold
rTMS stands for repetitive transcranial magnetic stimulation. The procedure can be carried out as outpatient and does not need anaesthesia. During the procedure magnetic impulses are delivered in pulses to focussed area of patient’s brain. These lead to involuntary twitching of skeletal muscles. Hence the intensity of rTMS pulse waves depending on the patient’s motor threshold (MT). MT is defined as the minimum stimulus strength required evoking skeletal muscle twitching at least five times out of ten. Usually the patient does not develop seizures.
 
5. C Obsessive compulsive disorder
Other indications include anxiety disorders and major depression. Psychosurgery requires informed consent and the approval of an independent review board. The patients must be 20 years or older and have a minimum of 3 years' disease duration with at least 2 years of unremitting symptoms despite active treatment. All patients must fulfil ICD-10 diagnostic criteria. Psychosurgery cannot be performed against the patient’s wishes under the Mental Health Act.
 
6. E The Camberwell Assessment of Need
The other measures include Medical Research Council Needs for Care Assessment, Cardinal Needs Schedule, The Avon Mental Health Measure and the Carers and Users Experience of Services. The Camberwell Assessment of Need assesses 22 domains of health and social need. It records staff and 33 patient views separately without giving primacy to either perspective. It has been translated into 22 languages and is the most widely used needs assessment internationally.
 
7. A An offence against someone with a mental disorder who is unable to refuse
There are three categories of offence against those with mental disorders.
  • Offences against patients who are ‘unable to refuse’
  • Offences against persons who are vulnerable and at risk of deception though have capacity to consent to sexual activity
  • Offences by care worker against patients with mental disorder
The legal definition of mental disorder in the act from the Crown Prosecution Service (2005) is the same as in the Mental Health Act 1983 (amended 2007).
 
8. D Patient should stop driving for at least 6 months and inform the DVLA as this is an isolated alcohol-related seizure
DVLA issues guidance to medical practitioners about the advice they should offer to patients. These are updated every six months and are freely available at www.dft.gov.uk/dvla/medical/ataglance.aspx. The guidelines related to alcohol or substance misuse mention that a person should stop driving immediately and inform DVLA, if he suffers from harmful use of alcohol or alcohol dependence. In this scenario, the patient has suffered an isolated alcohol-related seizure. Hence, he should stop driving and inform DVLA. His driving license would be revoked by DVLA for a period of at least 6 months. If he suffers recurrent seizures related to alcohol misuse, then epilepsy guidance related to driving should be followed.
 
9. C Inpatient suicide by hanging using a non-collapsible curtain rail
Never events are defined as ‘serious, largely preventable patient safety incidents’. The Department of Health has identified certain health care related events as ‘never events’. Occurrences of these events have serious implications for the service providers as these can be prevented by appropriate measures. Only one mental health-related incident is classified as a never event: death or serious harm to a mentally ill inpatient as a result of a suicide attempt using non-collapsible curtain or shower rails. This led to the use of collapsible rails in inpatient mental health wards in England and a reduction in inpatient suicide rates.
 
10. D 0.95
The experimental event rate (EER) is the event rate in the experimental, active or exposed group of patients. It is calculated by dividing the number of patients experiencing an event by the total number of patients:
EER: 190/200 = 0.95.
 
11. C 0.6
Clinical drug trials measure outcomes in a number of ways. First, and arguably the least clinically useful, the outcome measure is a symptoms rating scale – a continuous measure. From this, an effect size can be calculated. Second, and clinically most useful, it is a dichotomous measure such 34 as admission or readmission, dead or alive, relapses or non-relapses. In these studies, it is possible to obtain the control event rate (CER). This is simply the frequency of the event in question in the control group; it is calculated by dividing the number of control participants experiencing an event by the total number of control participants:
CER: 120/200 = 0.6.
 
12. C 0.35
The absolute benefit increase (ABI) is the absolute numerical difference between the rates of good outcomes between the experimental and control groups in a clinical study. The ABI is calculated from the difference between experimental and control event rates:
ABI = EER – CER = 0.95 – 0.6 = 0.35.
 
13. B 3
Usually the NNT is rounded off to the whole number, so the correct answer would be 3. The NNT measures the efficacy of a treatment. It is calculated from the reciprocal of the absolute benefit increase or absolute risk reduction. An NNT value <10 is regarded as clinically significant:
NNT = 1/CER – EER
= 1/0.35
= 2.85.
 
14. D Type 1 error
This occurs when a difference is found between two groups, although in fact there is no difference (false positive) and a null hypothesis is rejected. In other words, it is the false rejection of a null hypothesis when there is no true difference. It may happen as a result of bias or confounders. The probability of making a type 1 error is equal to the p value and is denoted by α. The power is the probability of demonstrating a significant difference between groups, where one exists. The standard deviation is a standardised measure of data dispersion. It is the spread of all observations around the mean and is calculated as the square root of the variance. The probability is the likelihood of any event occurring relative to a different number of possibilities.
 
15. E Type 2 error
This happens when the null hypothesis is accepted but is not actually true – when there is a small sample size and/or large variance. In other words, a type 2 error is the false acceptance of a null hypothesis when there is a true difference. The power of study is the probability of rejecting the null hypothesis when a true difference exists. Accuracy refers to the degree to which each research project’s methodology, instruments and tools are related to each other. It also measures whether research tools have been selected appropriately and whether the research methodology suits the hypothesis under investigation.
 
16. C Standard deviation
This is the average distance of observations from their mean. It is calculated by squaring each variation or deviation from the mean in a group of scores, then adding the squared deviations; this is then divided by the number of scores in the group minus 1, and the square root of the result is determined:
35SD = √∑(xx)² ÷ n – 1,
where x is the individual values, x the mean and n the total number of observations. Skewedness is a departure from the normal distribution. When the right-hand tail is extended, it is called a positive skew, and when the left-hand tail is extended a negative skew. The standard error is a measure of the uncertainty of a point estimate. It is in fact the spread or the SD of the sample mean. Confidence intervals are derived from the standard error.
 
17. E 99.7%
A normal distribution is also known as gaussian or bell shaped, and theoretically the mean, median and mode are equal. Approximately 68% of the population scores fall within first standard deviation of the mean, approximately 95% within 2 SDs and 99.7% within 3 SDs. Normally distributed data enable researchers to use parametric statistical tests, which have greater power than non-parametric tests. When variables follow a certain frequency over time or space, it is known as a Poisson distribution. When the result of a research study is a proportion, it is called a binomial distribution. In a t distribution, the continuous variable is measured and its mean difference between two groups estimated.
 
18. D Power
This is the ability to detect a significant difference between groups. It is also a measure of the strength of the relationship between two variables. The larger the sample size, the more power a researcher has to detect the difference. The power is also defined as the probability of rejecting the null hypothesis when a true difference exists. The type 1 error is α and the type 2 error β. The effect size is the difference between two means, which is divided by the standard deviation in controls.
 
19. A Mann–Whitney U test
In parametric tests, comparing two groups independently, the student’s independent t-test is used and, for paired groups, the student’s paired t-test. For comparing more than two groups, analysis of variance (ANOVA) is used. In non-parametric studies, comparing two unpaired and paired groups, the Mann–Whitney U test and Wilcoxon’s rank sum test are used, respectively. The Kruskal–Wallis analysis of variance is used for comparing more than two groups in non-parametric studies. In binary statistics, the χ² and McNemar’s tests are used for comparing unpaired and paired data, respectively. The χ² test is used to compare more than two groups in binary statistical tests.
 
20. A Analysis of variance (ANOVA)
Comparing more than two groups in parametric, non-parametric and binary statistical tests, the ANOVA, Kruskall–Wallis ANOVA and χ² test are used, respectively.
 
21. A ≥0.04
Probability is the chance of a type 1 error occurring. If the probability of making a type 1 error is equal to the p value then it is expressed as α. If α = 0.05, this means that there is only a 5% chance of erroneously rejecting the null hypothesis.
 
22. D −1 to + 1
The linear regression is the degree of relationship between two continuous, normally distributed variables that can be assessed using a linear correlation coefficient that ranges between −1 and 36 +1. This is a parametric correlation coefficient (Pearson’s ‘r’) and it can be plotted as a straight line. A correlation coefficient of 1 would indicate perfect positive correlation (i.e. both values rise together), whereas a value of −1 indicates perfect negative correlation. A correlation coefficient of 0 indicates that there is no relationship between the two variables.
 
23. B 4 ± 0.68
The confidence interval (CI) or confidence limit is an important measure of dispersion of data. Its properties fill the gap between analytical and descriptive statistics. It is defined as the range within which the true measure of the dispersion of data lies with a specific degree of certainty. One can be 95% certain that the true value of a measure such as a mean lies within a 95% confidence interval or two confidence limits of the estimate:
SE = SD ÷ √n = 1.4 ÷ √16 = 1.4 ÷ 4 = 0.35
The 95% CI ≈ mean ± 1.96 × SE
≈ 4 ± 1.96 × 0.35
≈ 4 ± 0.686
≈ 3.314 – 4.686
 
24. A Face validity
This measures the subjective assessment that the item appears to measure the desired quality. The criterion validity is the measure consistent with what we already know (gold standard) and what we expect. The concurrent validity is the extent to which the new measure relates to a different scale measuring something similar. The construct or predictive validity is the measure related to other variables as required by the theory. The convergent validity measures the degree of agreement between the measurements made using two different approaches, which are supposed to measure the same type of traits. The divergent validity measures the results obtained by an instrument that do not correlate too strongly with measurements of similar but distinct traits. It is useful to establish the construct validity of a construct/concept in a study by assessing whether it is different from other constructs in the proposed study.
 
25. B Construct validity
The predictive validity measures the extent to which a new measure predicts the outcome accurately. The convergent validity measures the degree of agreement between the measurements using two different approaches, which are supposed to measure the same type of traits. It therefore helps to establish the construct validity of a construct/concept in a study. The construct validity shows whether the findings gel with the theory. The content validity is the subjective assessment involved in the instrument sampling all the important contents of the attribute that is being measured.
 
26. B Internal consistency
The strength of the quantitative approach lies in its reliability. Internal consistency measures whether the score of the items correlates with the scores of all other items of the same construct. For internal consistency, a common approach is to calculate Cronbach’s α for continuous data. It is an average of all correlations between various items. It gives an estimate of the reliability of a psychometric test. Cronbach’s α (alpha aign) can be regarded as an extension of the Kuder–Richardson formula 20, which is used for dichotomous items. It is widely used in business, nursing, social sciences such as anthropology, archaeology, criminology, psychology, sociology, etc. The test–re-test or interrater reliability, measured at two different times with no treatment in between, should yield the same result. Its typical indices are k. The interobserver reliability measures the 37 agreement among interviewers who are rating the same information. The intraclass correlation coefficient is used in continuous data, and is useful when there is disagreement among various assessors in a research study.
 
27. A Berkson’s bias
This is a false association discovered through a survey of an unrepresentative sample. It is a subtype of sampling bias, also called admission bias. It is used in descriptive research and case–control studies. Researchers or participants can bring in the selection bias. The referral from a general practitioner to secondary care increases the concentration of rare exposures and severe diseases, which is an example of a ‘referral filter bias’. The exclusion of co-morbidity introduces a ‘diagnostic purity bias’ and makes the sample unrepresentative. Recruiting a member of an organisation may introduce a membership bias. Comparison of a new treatment in a current series of patients with an old treatment in a previous series of patients may be subject to historical control bias. When two groups of participants are enrolled in different ways and differ significantly as a result, it leads to an ‘ascertainment bias’.
 
28. C Interobserver reliability
This measures the agreement among raters who are rating the same information. Cronbach’s α is associated with internal consistency; k covers many similar measures of agreement in categorical data. It is a measure of diagnostic agreement among raters, which is corrected for the agreement expected by chance.
 
29. D Hawthorne’s effect
This is a non-specific effect caused by the participants’ knowledge that they are taking part in the study. The halo effect is the tendency of a rater to overestimate a participant’s response based on prior assumptions. People might try to please the experimenter as a goal and this may be called a demand effect. When a simulated medical intervention is given to the patient that may lead to perceived or actual improvement in a medical condition it is known as a placebo effect. The domino effect is defined as a chain reaction when a small change leads to another small change, and then a series of changes in a linear sequence. It can be described as a mechanical effect and is analogous to falling rows of dominos. Practically, it refers to a sequence of events linked to each other where time between successive events is relatively short. The domino effect can be considered both literally and metaphorically.
 
30. D Prevalence–incidence bias
This is a type of experimenter’s bias, also known a as Neyman’s bias and selective survival bias. It can occur in case–control and cross-sectional studies. In case–control studies, it is attributed to selective survival among the prevalent cases, but excludes mild, clinically resolved or fatal cases from the case group. Information bias is the general difficulty in assessing past exposures retrospectively. It arises as a result of misclassification of exposure to an agent being studied or misclassification of the disease or its outcome in a study. Publication bias refers to the phenomenon by which studies with positive results are more likely and studies with negative results less likely to be published. As the leading medical journals encourage reporting negative studies, it is becoming less of a problem. It can be evaluated using funnel plots and Galbraith’s plots. Confounding bias is an error in the interpretation that could be a correct finding.
 
31. A Clinical Practice Guidelines
Critically appraised databases are Clinical Practice Guidelines, Clinical Evidence, Evidence-Based Medicine and Evidence-Based Mental Health. They are systematically evaluated and developed 38 statements supporting clinicians and patients to make clinical decisions for special circumstances. They give explicit grade recommendations according to the quality of reviewed evidence.
Biological Abstracts, Cinahl, Cochrane library, EMBASE, PsycINFO and PubMed are electronic databases. There is little quality control and each database probably covers about 40% of the literature on a particular topic.
 
32. A Affective disorders
According to the current evidence omega-3 essential fatty acids are found to be beneficial in the prevention and/or treatment of unipolar or bipolar depression together with standard pharmacological treatment. The evidence for the treatment benefits for schizophrenia is less significant. They are also useful in various disorders that benefit from fatty acid supplementation, especially omega-3 fatty acids, e.g. rheumatoid arthritis. They provide protective benefit in heart disease and sudden cardiac death. They play a crucial role in brain functions, and normal growth and development, and they reduce inflammation. The main benefits of omega-3 essential fatty acids come from eicosapentaenoic acid (EPA) and docosahexaenoic acid (DHA), which are found in cold water fish, e.g. salmon, sardines, tuna.
 
33. B If x = 0.01, there is a probability of 1% that the null hypothesis is wrongly rejected.
The null hypothesis states that there is no difference or association between two or more groups of participants in a research study. In statistical terms, it is about disproving the null hypothesis rather than proving it. The factor x is the probability of a type 1 error. It is used to set the threshold for statistical significance, often arbitrarily as p = 0.01 – 0.05 (x = 1–5%), meaning that, if x = 0.01, there is a 1 in 100 or 1% chance that the true hypothesis is wrongly rejected.
 
34. A Mean > median
The mean is the average value derived by adding all the measurements, and then dividing the total by the number of measurements. The median is the middle value that divides the distribution into two equal halves. The mode is the most frequently occurring value in a data-set. When many observations are significantly higher than the median, we can assume that the mean of the distribution might be greater than the median. This translates to a positively skewed distribution.
 
35. A Allocation concealment
This refers to the process used to prevent prior knowledge of assignment before an allocation of participants has been completed. It seeks to prevent selection bias and also prevents the allocation sequence before and up to the assignment. Therefore, the investigators will not know the nature of the assignment of subsequent participants who enter the randomisation. To be properly randomised in a clinical trial, it should use a randomisation schedule that is not predictable. Blinding seeks to prevent ascertainment bias and protects the sequence after allocation. Matching refers to allocating the participants to treatment groups according to set criteria to achieve uniform groups across the board. Blinding and masking are interchangeable terms and they refer to the fact that the participants and/or investigators are unaware of the study treatment.
 
36. C Q statistics
In a meta-analysis, the results of different studies are combined because individual studies are often too small to yield reliable treatment effects. Although similar studies are included in the meta-analysis, there is a possibility that each study differs from all the others just by chance. Sometimes, the differences can be due to foreseeable factors such as dose of drugs tested, mean 39 age of the study population, different rating scales, etc. To measure whether this heterogeneity is more than random (expected), it is necessary to conduct certain tests of heterogeneity. These include Q statistics (e.g. χ2 test), which tests the null hypothesis of homogeneity. A χ2 test measures the amount of variability due to heterogeneity. Galbraith’s plot and the l’Abbé plot are pictorial representations of heterogeneity. The paired Student’s t statistic is useful for examining differences in the means of two populations consisting of independent samples, continuous data or at least interval and normally distributed data with equal variances. Stratification is the process of allocating participants in a study population to homogeneous groups before sampling takes place.
 
37. B Clinical significance of the results
Randomised controlled trials are conducted under strict protocols, which include inclusion and exclusion criteria. The study population (highly homogeneous) may not represent the general population, which is often encountered in clinical practice. Therefore, there is poor generalisability of research findings to daily clinical work.
 
38. A Taking amphetamines during pregnancy does not increase the risk of ADHD in the child.
If the psychiatric trainee wants to prove that pregnant mothers taking amphetamines increase the risk of ADHD in their offspring, it is necessary to consider the null hypothesis. It is best to assume that maternal consumption of amphetamines does not increase the risk of ADHD and then proceed to disprove it. The converse of the null hypothesis is known as the alternative hypothesis.
 
39. B Identifying a suitable control group
In a cohort study, there is no need to randomise the participants, so there is no need to conceal allocation. It is often most difficult to identify a reasonable control group that lacks the exposure of interest of the study. Using valid instruments and reasonable follow-up, it is easy to analyse the findings.
 
40. B 2.5
In the above study, the control event rate (CER) is 20%. The experiment event rate (EER) is 70%.
The absolute rate reduction (ARR) is the difference between the two event rates, i.e. 70 – 20 = 50%
The relative risk reduction (RRR) = 50/20 = 2.5. In other words, the ARR is the absolute numerical difference between the rates of adverse outcomes between the study and the control samples.
The RRR, i.e. the relative benefit increase, is calculated as follows:
RRR = ARR.
 
41. A 2
Two patients must be treated with aripiprazole to have one additional response. The NNT can be calculated from the absolute risk reduction (ARR):
NNT = 1/ARR = 1/0.5 = 2.
 
42. B 9.3
The odds ratio refers to comparison of the odds of an event happening in a study group with that happening in another group, e.g. a control group. To calculate the odds ratio, it will be useful to 40 construct a 2 × 2 table. As per protocol, analysis is used, except that those patients who completed the study will be included in the analysis (Table 1.2).
Table 1.2   A 2 × 2 table
Study arm
Response
No response
Total
Aripiprazole
70 (a)
30 (b)
100
Placebo
20 (c)
80 (d)
100
The odds ratio is calculated using the cross-product ratio
ad/bc = (70 × 80)/(30 × 20) = 5600/600 = 9.3.
 
43. E The direction of the effect cannot be determined.
Cross-sectional studies simultaneously ascertain the presence or absence of both disease and an exposure at a particular point in time. They are effectively studies of prevalence or frequency of disease. They cannot reliably distinguish cause and effect. The direction of the effect cannot be determined because the risk and outcome are being studied at the same time. The other factors are less of a problem or similar in difficulty for other study designs as well.
 
44. C Systematic reviews of RCTs are above systematic reviews of cohort studies.
The evidence must contain an explicit and reproducible method for weighing the results according to quality and precision. In formulating clinical practice guidelines, a well-conducted meta-analysis with little or no heterogeneity of RCTs is accepted as the highest recommendation.
Hierarchy of evidence for clinical practice guideline recommendation
Level of evidence clinical studies
IA
IB
IIA
IIB
III
IV
Systematic reviews and meta-analyses of RCTs
RCTs
Well-designed non-RCTs
Other well-designed quasi-experimental studies
Evidence from well-designed non-experimental studies
Evidence from expert committee reports and respected authorities
 
45. A Individual RCTs
A randomised treatment trial of two types of family interventions reported on anorexia nervosa. The study included ‘conjoint family therapy’ (CFT) and ‘separated family therapy’ (SFT). A stratified design was used to control for levels of critical comments using the expressed emotion index. The researchers undertook both forms of therapies with separate supervisors. Marked symptomatic improvement was noticed in the SFT group and psychological change in the CFT group.
 
46. A Individuals show improvement because they are aware of their participation in research.
Individuals improve just because they are being studied. Smaller sample sizes giving false-positive results are called type 2 errors. The other answers are not described by any particular phenomenon.
 
4147. A It stops the selection biases from creeping back into the study.
Intention-to-treat analysis helps to reduce bias. In interventional studies, there are possibilities of participants dropping out or moving from one arm to another for various reasons. It takes into account all these participants and analyses them based on the original study protocol. It prevents selection biases from creeping back into the study and helps to avoid overestimating the benefits of treatment.
 
48. D Replication studies on a different population help to add weight to the findings.
Such studies in different populations and different parts of the country add weight to the original findings. Ethical approval would still be required and may not be any easier to get than for an original study. Although it may be advantageous, it is not mandatory to use exactly the same methods as the original study. There is a marked publication bias because studies with positive results tend to be reported and published more often than those with negative results. In studies using humans we can never be certain about how representative the sample was. It is therefore always important to repeat studies with different samples in different parts of a country or the world.
 
49. C Research with positive results has a higher chance of being published.
Publication bias is implied when positive studies tend to be reported and published more often than negative ones. It is not uncommon to find a similar study design being carried out at different centres, because work done with negative results hardly ever gets published. This is not deliberate suppression of data, but an understandable desire for researchers and journals to publish positive findings.
 
50. D It is also known as a dichotomous variable.
A variable is something that we measure, manipulate and control in an experiment. There are different ways of describing and differentiating variables. A binary variable takes only two possible values and helps to describe binary data with only two mutually exclusive categories. A variable that can take any value within a limited range is called a continuous variable. A variable can be converted into a binary variable, e.g. by defining a cut-off. A dependent variable is a variable that is dependent on the independent variable – also called an outcome variable. An independent variable is a variable that is being examined in an experiment to identify its effect on the dependent variable – also called an experimental or predictor variable. In non-experimental research, the researchers do not manipulate the independent variable because it might be unethical or impractical to do so.
 
51. A It was conducted in three academic centres between 1997 and 2002.
POTS was conducted in three academic centres across the USA. The main objectives were to evaluate the efficacy of cognitive–behavioural therapy (CBT) alone, sertraline alone, and CBT and sertraline combined as an initial treatment for children and adolescents with obsessive–compulsive disorder (OCD). The duration of treatment was 12 weeks and the participants were recruited between September 1997 and December 2002. The findings of the study suggested that children and adolescents should start treatment with a combination of CBT plus an SSRI or CBT alone.
 
4252. A All-cause discontinuation was the primary outcome measure.
Compared with other antipsychotics, quetiapine was less potent in treating the illness. Of the three antipsychotics compared, risperidone was associated with the smallest elevations in fasting triglyceride and cholesterol levels. The overall PANSS scores were comparable in all three arms; however, greater reduction in a positive symptom subscale was persistently seen in the olanzapine arm. Safety and tolerability were a secondary measure.
 
53. B Cost Utility of the Latest Antipsychotics in Schizophrenia Study 1
CUtLASS 1 stands for Cost Utility of the Latest Antipsychotic Drugs in Schizophrenia Study 1. It was a randomised controlled trial of the effect on quality of life of second- versus first-generation antipsychotic drugs used to manage schizophrenia. It was conducted by the Institute of Psychiatry in London and was funded by the NHS.
 
54. D One of the outcome measures included scores of quality-of-life scales.
Outcome measures included quality-of-life scores, symptoms, adverse effects, participant satisfaction and costs of care. Clozapine was considered in CUtLASS 2, not in this study. Depot preparations of the first-generation antipsychotics were taken into account. Some 227 participants from 14 community psychiatric services in the NHS were randomised.
 
55. C Qualitative research
The best way to achieve a deeper understanding of a particular clinical issue is through qualitative studies. They help to generate ideas, opinions and hypotheses. If they are conducted well, they offer insight and are valid. They can also inform and improve the quality of quantitative research. However, they are prone to researchers’ bias and lack reliability if only one researcher was involved. They can illuminate complex issues. If we want to find out the incidence or check the cause-and-effect hypothesis for an exposure and outcome, we can use case–control and cohort studies, which are mainly observational studies. To find out which treatment or intervention is better in a population, we should use RCTs. Cross-sectional studies are helpful in finding out the prevalence.
 
56. B Cost-effectiveness analysis
This is a form of economic analysis that compares the cost of an organised treatment/intervention or activity with its effectiveness, measured in terms of natural health outcomes, e.g. life-years gained. It means that, if there is more than one strategy with similar results, cost-effective analysis is used to identify the most cost-effective strategy of the lot. It is usually expressed as the cost per health outcome, e.g. the amount earned by preventing cases of polio through use of the vaccine compared with the cost of polio vaccine itself.
 
57. B External validity
This term is used to describe the quality of a systematic review. It refers to the degree to which conclusions of one study would hold together different people in different places and at different times. Assigning weight to trials in a systematic review involves looking at methodological quality, precision and external validity, as well as other features.
 
4358. B Catatonia
The Bush–Francis rating scale is used to assess catatonia. It is available as a 23-item scale as well as a shorter version, consisting of 14 items. The Simpson–Angus scale and abnormal involuntary movements scale (AIMS) are useful in assessing extrapyramidal side effects. Catatonia is characterised by marked psychomotor retardation, which may be accompanied by motoric immobility or excessive motor activity, mutism, excessive negativism, echolalia, echopraxia or abnormal voluntary movements. Malignant catatonia (replacing the term ‘lethal catatonia’) is characterised by autonomic instability and hyperthermia. Catatonia is usually associated with schizophrenia and mood disorders, and less frequently with neurological, endocrine and metabolic disorders, infections, and drug withdrawal and toxicity. Most catatonic patients respond to benzodiazepine (especially lorazepam) within 3–7 days. ECT is helpful in catatonia presenting in schizophrenia and mood disorders.
 
59. B Depressive disorder with psychotic symptoms
Depressive mood preceded the psychotic symptoms. In this case, the psychotic symptoms are mood congruent and not of definite schizophrenic type (ICD-10 criteria a–d of schizophrenia). Therefore, this is more likely to be a depressive episode with psychosis. The patient does not fulfil the ICD-10 criteria for schizophrenia, post-schizophrenic depression or schizoaffective disorder. There is no evidence of mania.
 
60. D Prisoners awaiting trial
Ganser’s syndrome is characterised by sudden, abrupt ending, ‘approximate answers’ (Ganser’s symptom, vorbeireden), clouding of consciousness and amnesia of the episode, auditory and visual hallucinations or pseudo-hallucinations, and dissociative symptoms. In the choice of answers, the patient appears deliberately to pass over the indicated correct answer to select a false one, which any child could recognise as such. It is a rare phenomenon and is more likely to be seen in those prisoners who are awaiting trial. It has been thought of as schizophrenia, an acute psychotic reaction or factitious/simulated psychosis. It is treated as an acute schizophrenic episode.
 
61. B Impaired glucose tolerance
In anorexia nervosa, cortisol-releasing hormone, cortisol, growth hormone and reverse triiodothyronine (T3) levels are increased whereas LH, follicle-stimulating hormone (FSH), somatomedin C, T3 and C-peptide are decreased. Anorexia nervosa is also associated with hypokalaemic and hypochloraemic alkalosis caused by vomiting. Insulin release is delayed and glucose tolerance impaired. It is important to differentiate anorexia nervosa from weight loss due to other causes and the above changes will help.
 
62. D Family therapy
This scenario appears to be a case of high expressed emotions within the family environment. Family therapy including psychoeducation of family members would be helpful. CBT will not be of any help in this scenario because there are no cognitive distortions and the patient is already making good progress. For the same reasons, the community treatment order under the Mental Health Act is also not indicated because the patient has engaged with the services. Interpersonal therapy is less likely to be helpful, and is mainly indicated in depression. Counselling could be helpful if family therapy is not available.
 
4463. A Disorganised schizophrenia
This is characterised by disorganised speech, disorganised behaviour, and flat or inappropriate affect. It is classified in DSM-IV but not in ICD-10. DSM-IV includes paranoid, disorganised, catatonic, undifferentiated and residual subtypes.
ICD-10 also includes paranoid, catatonic and simple schizophrenia subtypes. Post-schizophrenic depression is characterised by the presence of significant depressive symptoms of at least 2 weeks’ duration in the aftermath of schizophrenia, but in the presence of some schizophrenic symptoms, which do not dominate the clinical picture.
 
64. E Simple schizophrenia
This is characterised by insidious and progressive deterioration for at least 1 year when the patient is unable to meet the demands of society and declines in total performance. Negative symptoms appear without any prior psychotic symptoms. Delusions and hallucinations are not evident in these patients. Asperger’s syndrome, autism and schizoid personality disorder would be present from childhood and continue into adulthood. Catatonic schizophrenia is dominated by psychomotor disturbances that alternate between stupor and hyperkinesia or automatic obedience and negativism.
 
65. D Family history of depression
Prognostic factors were carefully identified by Valliant and others for patients with schizophrenia before operational definitions were used. Valliant’s predictors of a good prognosis in schizophrenia are less central to the disorder, and include: an acute onset of illness, the presence of a stressful precipitating event, a family history of depressive illness, prominent affective symptoms, an absence of family history of schizophrenia or schizoid premorbid personality traits, and confusion or perplexity. Overall, the outcome seems to be determined more by the circumstances under which the illness develops and the premorbid personality than by the clinical features of the illness. Various studies have observed that, of patients with first-episode schizophrenia, 50% improve, 20% experience no further episode, 40% obtain employment/higher education and 20% live independently.
 
66. C A fugue state is usually associated with a period of wandering, for which there is a subsequent amnesic gap on recovery.
The fugue state is a syndrome in which there is a sudden loss of all autobiographical memories and knowledge of personal identity, usually associated with a period of wandering, for which there is a subsequent amnesic gap on recovery. Fugue states differ from transient global amnesia or transient epileptic amnesia in that the person does not know whom he or she is. Fugue states are almost always preceded by a severe precipitating stress. Depressed mood is also an extremely common antecedent for a psychogenic fugue state.
 
67. B It is found more commonly post mortem in people with alcohol problems than it is diagnosed in life.
Korsakoff’s syndrome is caused by thiamine deficiency. It can arise due to medical conditions. However, the most common cause is chronic and persistent alcohol misuse. This leads to retrograde amnesia and confabulation. In many patients with atypical symptoms, the clinical diagnosis could be easily missed. The confirmation is made post mortem, when damage to mammillary bodies and anterior thalamic nuclei is shown.
 
4568. A Herpes encephalitis can give rise to a severe form of amnesic syndrome.
Most cases are said to be due to primary infections, although there may be a history of a preceding ‘cold sore’ on the lips. Usually there is a fairly abrupt onset of acute fever, headache and nausea. There may be behavioural changes and seizures can occur. The fully developed clinical neuropathological and neuroimaging studies show that there is extensive bilateral temporal lobe damage.
 
69. A Absence of voluntary control on facial expressions
Pathological laughter and crying (PLC) is defined as uncontrollable laughter, crying or both in the absence of an associated affect. It is also known as a pseudo-bulbar affect or emotional incontinence. It is a primary disorder of emotional expression and not a disorder of feelings. It occurs due to non-specific stimuli and there is no corresponding mood change. In fact, it is associated with feelings of happiness and sadness. PLC is probably caused by loss of the voluntary inhibition of the laughter and crying centre, presumably located in the upper brain stem. It has been noted in a variety of neurological conditions, e.g. galactic epilepsy, multiple sclerosis, pseudo-bulbar palsy, cerebellopontine tumours, trigeminal neuralgia and petroclival meningioma. Amitriptyline 75 mg has been found to be effective. Fluoxetine, sertraline, amantadine or levodopa can also be used.
 
70. A The patient can present with subcortical dementia.
Cognitive impairment is likely to be subcortical dementia because multiple sclerosis involves mainly the white matter. Mania can occur as a part of the physical disorder or secondary to medication. The patients who have a premorbid or family history of affective disorders are at a higher risk of hypomania when treated with steroids. Hence, in this case caution should be exercised. Early onset of illness is associated with increased risk of suicide. Lifetime prevalence of depression is anywhere from 25% to 50%. The co-occurrence of psychosis and multiple sclerosis is not that common.
 
71. A Chronic fatigue syndrome
The US Centers for Disease Control and Prevention (CDC) criterion for chronic fatigue syndrome is: new onset of severe unexplained fatigue lasting for more than 6 months that is neither related to exertion nor relieved on rest. For the patient this is a functional impairment. In addition, the patient presents with at least four of symptoms such as impaired memory, concentration, sore throat, tender lymph nodes, pain in the joints and muscles, headaches, poorly refreshing sleep and post-exertional malaise for more than 24 hours. Evidence continues to mount to suggest that this syndrome is a discrete post-infectious entity and not simply a variant or disguised mood or anxiety disorder.
 
72. C Genetic constitution increasing vulnerability to schizophrenia
Among the predictors of schizophrenia, the most interesting concept is schizotaxia, which is used to describe the genetic vulnerability to schizophrenia. It presents as an aggregation of subtle symptoms of brain dysfunction expressed, in part, as negative symptoms, neuropsychological deficits, including working memory/attention problems, problems in long-term memory and concept formation/abstraction, but not as a psychosis. This syndrome is qualitatively similar, although less severe, not fulfilling the criteria for schizophrenia. A study suggested that the symptoms of schizotaxia occur in 20–50% of adult relatives of patients diagnosed with schizophrenia, although genetic liability to develop schizophrenia does not inevitably lead to schizotypal personality disorder, schizoid personality disorder or schizophrenia.
 
4673. B Epilepsy–parasomnias
These sleep disorders are a category of sleep disorders that occur at various stages of sleep. They consist of abnormal and unusual movements, behaviours, emotions, perceptions and dreams. They are classified into two major types: non-rapid eye movement (NREM) parasomnias and rapid eye movement (REM) parasomnias. NREM parasomnias include sleep walking/somnambulism, night terrors, bruxism and restless legs syndrome. REM parasomnias include REM sleep behaviour disorder, recurrent isolated sleep paralysis and cathrenia, which consists of breathholding and expiratory groaning during sleep. The following are the correct associations for the other disorders:
  • Dementia–sun downing
  • Huntington’s disease–frequent awakening
  • Kleine-Levin syndrome–hypersomnia
  • Parkinsonism–frequent awakening, disturbed circadian rhythm.
 
74. A Delusions involving the skin
Somatic delusions may present with a delusional belief of skin infestation, in which the patients believe that they have organisms/parasites (usually worm like) crawling over or into the skin/nails. Occasionally they may believe that the bodies are inanimate, similar to seeds. The ‘match box’ or ‘pill box’ sign is seen in delusions involving skin where a patient comes up with a small container that has ‘insect corpses’ or ‘eggs’, which are usually skin scrapings, dry mucus or possibly pieces of lint.
 
75. D Overall, the best-attested treatment result refers to somatic subtype.
After starting therapy with an antipsychotic, it takes about 2 weeks before the delusional system starts to ameliorate. If the medication is stopped, the delusions return within days or weeks. Recovery is often good but slow and the illness may continue for a very long time. The patients may need to continue with medication on a long-term basis. The maintenance dose is usually very small. Overall the best-attested treatment result refers to somatic subtype and there is virtually no literature on the grandiose type.
 
76. E Pseudo-dementia
EEG may aid the differentiation between degenerative brain disease and depressive pseudo dementia. In EEG, there are four components: δ (<4 Hz), θ (4–7 Hz), α (8–13 Hz) and β (>13 Hz) wave. Pronounced flattening of the EEG may raise the possibility of Huntington’s disease. Creutzfeldt–Jakob disease has repetitive spike discharge or triphasic sharp wave complexes. In the early stages of Alzheimer’s disease, the EEG is nearly always non-specifically abnormal. However, in frontotemporal degenerations, the EEG remains unaffected. In presenile Alzheimer’s disease, a waves disappear and show irregular θ waves with superimposed runs of δ waves.
 
77. D Personality change and incongruous behaviour
The most common psychiatric presentation in Wilson’s disease is personality change, which is found in 25% of patients. In another 25% incongruous behaviour is noticed and is usually the reason for referral. Mild cognitive impairment is suspected in around 25%, depression in 21%, psychosis in 2% and disorientation in 7%, irritability in 18% and aggression in 14%. Wilson’s disease is also called hepatolenticular degeneration. It is an autosomal recessive disorder in which copper builds up in the body, especially the liver and brain. The faulty gene ATP7B on chromosome 13 causes it. 47 The build-up of copper in the cornea causes Kayser–Fleischer rings – a brownish pigmentation. Wilson’s disease leads to anaemia, kidney and heart problems, pancreatitis, menstrual disorders, miscarriages and premature osteoporosis.
 
78. D They show a tendency to appear in the later stages of the disease.
Unfortunately, the neuropsychiatric features of SLE are quite varied and lack any particular form or pattern. Mental disorders are the most common neuropsychiatric manifestations. Most of them are usually transient and clear up within 6 weeks and rarely within 6 months. However, they are recurrent and tend to develop when other systemic features relapse. Headache is perhaps the most common symptom of neurological manifestations, often with migraine features. Other neuropsychiatric manifestations include cognitive impairment, mood and anxiety disorders, psychosis, seizures, polyneuropathy and cerebrovascular disease.
 
79. E Paranoid features figure prominently in psychosis.
The psychosis in myxoedema shows features of delirium, florid delusions and hallucinations. Auditory hallucinations are particularly common. There is a profound loss of interest; mental sluggishness and mood change indicate apathy rather than depression. The memory is affected from the early stages and person is forgetful of day-to-day things. Myxoedema may present with prominent paranoid features. Depressive psychosis does not respond readily to treatment by psychotropic medications until myxoedema has been treated successfully.
 
80. A Depression is the most common psychiatric presentation in Cushing’s syndrome due to pituitary involvement.
However, this is not the case in Cushing’s syndrome caused by direct adrenal involvement, e.g. adrenal adenoma or carcinoma. Psychiatric disturbances are found in more than 50% of patients. Severe psychosis often presents with accompanying depression, paranoia and auditory hallucinations with marked fluctuations. Physical changes include moon face, buffalo hump and purple striae on the abdomen with early weight gain. A plethoric complexion with hirsutism, excessive bruising, skin pigmentation and hypertension occurs with mild glycosuria.
 
81. B It follows a more severe course when the onset is early.
In Huntington’s disease (HD), the ratio of incidence in males and females is equal. It rarely skips a generation in its presentation. Choreoathetoid movements usually start in the face, hands and shoulders, and may lead to a dance-like ataxia. Hemichorea may also be seen. Some patients develop striate rigidity rather than chorea, especially when the onset is at an early age. This variant is called the ‘Westphal variant’ which is associated with tremor, akinesia and cogwheel rigidity, and at times progresses to torsion dystonia. Fits are more common in this variant (16% compared with 3% in general HD). Dementia follows involuntary movements. Recall is particularly impaired, and slowing of cognition is generally marked from the start. Rigid thinking with an inability to change from one subject to another is impaired. It follows a more severe course when the onset is early. Dysarthria often presents early in the disease. The abnormal movements cease during sleep. Psychiatric changes usually occur years before the diagnosis is fully established. There is a change of disposition, emotional lability and paranoia. Depression is usually severe with a high risk of suicide in the early stages (accounts for 7% deaths in non-hospitalised patients). It usually responds to medication or ECT. If psychosis occurs first, delusions are usually followed by hallucinations (delusional–hallucinatory).
 
4882. B Life events are common in the 6 months before an act of parasuicide.
Parasuicide has been defined by Norman Kreitman as any act deliberately undertaken by a person who mimics the act of suicide, but that does not result in a fatal outcome. It is a self-initiated and deliberate act in which the person injures him- or herself, or takes a substance in a quantity that exceeds the therapeutic dose (if any) or his or her habitual level of consumption, and that he or she believes to be pharmacologically active. Most cases of parasuicide are associated with a psychiatric disorder, the most common being: depressive episode, dysthymia, alcohol dependence, personality disorder, etc.
Life events are more common in the 6 months before an act of parasuicide, including:
  • Break-up of a relationship
  • Being in trouble with the law
  • Physical illness
  • Illness of a loved one
  • Predisposing factors include:
  • Marital difficulties
  • Unemployment
  • Physical illness
  • Mental impairment
  • Death of one’s parent at a young age
  • Parental neglect.
 
83. D Studies of college students have suggested that a quarter meet DSM-IV criteria for BDD.
This is a type of somatoform disorder in which there is a disturbance of body image, and preoccupation with an imagined or slight abnormality in appearance. The person’s concern is grossly excessive for a slight physical anomaly. The belief is not of delusional intensity although in clinical practice this distinction is often difficult. Anorexia nervosa fulfils the diagnostic criteria for BDD. The most common age of onset is from adolescence through to the third decade. In psychodynamic theory, BDD has been considered to represent an unconscious displacement on to body parts of sexual or emotional conflicts, feelings of inferiority, poor self-image or guilt. Its co-morbidity includes depression, social anxiety disorder, obsessive–compulsive disorder, agoraphobia, avoidant personality disorder, borderline personality disorder and dependent personality disorder.
 
84. A About a quarter of new consultations in secondary care will be for medically unexplained symptoms.
This means that doctors treating patients with these symptoms have not found a medical cause, but this does not mean that a medical cause does not exist. These symptoms may be considered synonymous with somatisation disorder. Psychiatric co-morbidity is common in these patients, but the symptoms may not entirely be explained by psychiatric factors. In any week, 60–80% of healthy people experience bodily symptoms, but only a small portion of these will see their GP. About one in five new consultations in primary care will have medically unexplained symptoms. Up to 40% of patients seen by hospital medical specialists have medically unexplained physical symptoms and receive no organic diagnosis.
 
85. E 40–65%
The percentage risk of developing schizophrenia depends on the relationship to the person with schizophrenia:
  • 49Identical monozygotic twins: 40–65%
  • Fraternal twins: 0–15%
  • Children of both parents with schizophrenia: 40%
  • Children: 13%
  • Parents: 6%
  • First cousin: 2%.
 
86. E Urinary incontinence can occur in this condition.
Pseudo-seizures are paroxysmal episodes that resemble epileptic seizures. They are often misdiagnosed as epileptic seizures. Of patients with refractory seizures, 20–30% are considered to have pseudo-seizures. About a third (10–30%) of patients with pseudo-seizures have coexisting epilepsy. It is difficult to differentiate between true seizures and pseudo-seizures by clinical observation alone. During the episode of a pseudo-seizure, patients can present with tongue bite, urinary incontinence or falls, which are generally absent. In the post-ictal phase, the gag and papillary reflexes are retained and there is an increase in prolactin levels. Pseudo-seizures are psychological in origin but can be organic, e.g. syncope, migraine and transient ischaemic attacks. The most conclusive test for differentiating pseudo-seizures from epilepsy is long-term video-EEG recording. Psychiatric disorders that resemble pseudo-seizures include factitious disorder, malingering, panic disorder, schizophrenia and depersonalisation disorder.
 
87. A Hyperreligiosity
The Gastaut–Geschwind syndrome is associated with involvement of temporolimbic lobe and temporal lobe epilepsy. It can present with some behavioural problems such as hyperreligiosity, exaggerated philosophical concern, hypergraphia, hyposexuality, circumstantiality, intense mental life, or interpersonal ‘stickiness’ or viscosity. It can also present with increased violence and hostility in the interictal period.
 
88. A Better response to antipsychotics
Late-onset schizophrenia is characterised by more positive symptoms, fewer negative symptoms and less affective blunting. Patients have a less formal thought disorder, better neuropsychological functioning and better response to antipsychotics. Many of these patients need low-dose antipsychotics to have good symptom control.
 
89. D Recurrent depression
A literature review demonstrated the strong link of depression with cardiovascular disease. This was observed in patients of all age groups.
 
90. D Increasing age
Seizure threshold is affected by many factors. It increases with age and is higher in men than in women. Seizure threshold is decreased by most antidepressants and antipsychotics, whereas benzodiazepines and anticonvulsants increase it. Seizure threshold increases during the course of ECT but returns to baseline within a few months of stopping it.
 
91. C Parents with a forensic history
There are several wide-ranging factors associated with development of conduct disorders in children. Table 1.3 describes some of the common associations.
50
Table 1.3   Factors associated with conduct disorders in children
Association
Disorders
Inner city, isolated communities
Increased risk of all child psychiatric disorders
Low maternal education
Externalising disorders, separation disorder
Parental sociopathy
Conduct disorder and oppositional defiant disorder
Family dysfunction
Conduct disorder, hyperactivity, emotional disorders
Mothers with internalising disorders
Children with externalising disorders, e.g. ADHD
 
92. B Figures of overall prevalence are similar across cultures around the world.
The community prevalence of psychiatric disorders is 2–30%. This reduces to 12–15% when only moderate-to-severe diagnoses are considered. The proportion of girls with psychiatric diagnoses relative to boys increases with age, but overall prevalence is higher for boys. The figures for overall prevalence of childhood and adolescent mental disorders are similar across cultures around the world.
 
93. A Childhood depression is associated with higher rates of depression in adult life.
The point prevalence of major depression and dysthymia in children is 2%, and 2–5% for adolescents. Depression is equally common in boys and girls up to adolescence. There is a high rate of recovery but a high rate of relapse in children with depressive illness. Childhood or adolescent depression is associated with higher rates of depression in adult life.
 
94. D Teaching begins at the clinic and then is gradually taken home.
This was developed by Eyberg at the University of Florida. The therapy is delivered to teach parents to build a warm relationship with the child and to teach the child how to behave appropriately. The aim of the therapy is to improve the quality of child–parent relationships, and consists of two phases. The therapist sits behind a one-way mirror and guides the parents with a microphone.
 
95. E Risperidone
Antipsychotic drugs are the most widely studied drugs in autistic spectrum disorders. They act by dopamine receptor blockade and help to reduce maladaptive behaviours such as stereotypies and self-harm. Selected medications for use in domains of behaviours in autism include:
  • Stereotypical and compulsive/repetitive behaviours: SSRIs (fluoxetine/fluvoxamine) and atypical antipsychotics (risperidone)
  • Irritability, aggression and self-injury: atypical antipsychotics (risperidone), opiate antagonist (naltrexone) and typical antipsychotics (haloperidol)
  • Hyperkinesia and inattention: methylphenidate, α2-receptor agonist (clonidine) and atomoxetine
  • Depressive symptoms: SSRIs
  • Manic symptoms: anticonvulsant mood stabiliser (valproate/divalproex)
  • Anxiety symptoms: SSRIs and buspirone
  • 51Sleep dysfunction: melatonin and an antihistamine
 
96. B Family therapy
NICE guidelines on depression give guidance on the management of depression based on its severity. Mild and moderate depressive episodes are mainly treated (first line) with psychological therapies, i.e. cognitive–behavioural therapy (CBT), interpersonal therapy or family therapy. In severe depressive episodes, medication is the first-line treatment. However, we should be careful in choosing therapy for children and adolescents.
 
97. A 13%
There was a report that 13.2% of the young people questioned in a trial had tried to harm themselves at some point in their lives, 6.9% in the previous year; 15% had reported thoughts of suicide and 54% of those reporting self-harm described more than one episode/incident. The presentation to hospital was described in only 12.6% of adolescents who had engaged in self-harm.
 
98. D Separation anxiety
School refusal can be the presenting complaint for children with a variety of underlying disorders, with separation anxiety being the most common diagnosis, particularly among younger children. In many cases, school refusal results from the combination of a child who does not want to separate and parents who do not insist on school attendance. This is because either they have difficulty imposing limits in general or they share their child’s anxieties about separation.
 
99. A Citalopram
Treatment of severe depression in children presents a challenge because there are limited options available compared with the adult population. NICE guidelines provide useful information to meet this challenge (Table 1.4).
Table 1.4   Pharmacological treatment for depression in children and adolescents: UK and US guidelines and licensing of antidepressants
Guideline advice in terms of pharmacological treatment
Minimum age for licensed prescribing (years)
Drug
NICE
Texasa
UK
USA
Citalopram
Second line
First line
18+
18+
Escitalopram
Not discussed
Second line
18+
12+
Fluoxetine
First line
First line
8+b
8+
Paroxetine
Contraindicated by the CSM
Second linec
18+
18+
Sertraline
Second line
First line
18+
18+
Venlafaxine
Contraindicated by the CSM
Third line
18+
18+
CSM, Committee on Safety of Medicines; NICE, National Institute for Health and Care Excellence; Texas, Texas Children’s Medication Algorithm Project. aFor major depression of sufficient severity to warrant medication. bFor moderate-to-severe depression. cAdolescents only.
 
52100. E About 10% of autistic individuals go through a phase in adolescence when they lose language skills.
Autistic individuals are not at an increased risk of developing schizophrenia in adult life. Even with IQ and speech on their side, autistic children have only around a 50% chance of a good social outcome in adult life. Autistic aloofness tends to improve with time in just over half of all cases. Autistic children who do not have useful speech by age 5 rarely go on to develop it. Sometimes, there is intellectual deterioration as well. The decline is not progressive, but the lost skills are not generally regained.
 
101. B Conduct disorder
Most psychological traits have been found to have a heritability of around 40–60%. This means that genetic differences between individuals account for roughly half the observed variance in a given population. Conduct problems are the one probable exception to this rule, with most (but not all) studies showing a small genetic contribution. Conduct disorders commonly cluster in families but, compared with other child psychiatric disorders, shared environment has a proportionally greater influence than shared genes. Although twin studies have shown high concordance for monozygotic pairs, the concordance for dizygotic pairs is also high. Adoption studies have shown the influence of biological parents to be less than that of the adoptive ones. However, they show a strong interactive effect whereby a difficult temperament (indexed by antisocial behaviour in biological parents) and unfavourable environment (as indexed by antisocial behaviour in the adoptive parents) leads to a far higher rate of antisocial behaviour and criminality than would be expected by addition.
 
102. E Parent management training
This is the best-established approach for managing conduct disorders, with numerous randomised controlled trials attesting to its effectiveness. It gets parents to pay attention to desired behaviour; positive aspects of child–parent relationships are promoted and parents are taught effective techniques for handling undesirable behaviour. It can be given more economically in groups while maintaining effectiveness.
 
103. A Down’s syndrome
This is due to full trisomy 21 (47,XX +21 or 47,XY + 21) in 95% of people. Robertsonian translocation between acrocentric chromosomes accounts for another 5%. The trisomy occurs from anew. Shuttleworth made two interesting observations in 1909: Down’s syndrome children tended to be the youngest of their siblings and Down’s syndrome was associated with advanced maternal age. Non-disjunction is the common mechanism. Down’s syndrome is the most common example of a genetic condition that is not inherited. It occurs in 1:600–1:1000 newborns.
 
104. A Men with Down’s syndrome due to non-disjunction have a high incidence of infertility.
The development of sexuality is an issue for patients with Down’s syndrome. Down’s syndrome girls start menstruating at almost the same age as other girls and have ovulatory cycles. Women with Down’s syndrome are known to have had children. However, the fertility rate is significantly lower than in the general population, probably due to slow follicular development in the ovaries, and complete failure to ovulate in some women. Menopause occurs on average 5 years early in these women. Most of the men have reduced sperm counts due to unknown causes. Men with mosaic Down’s syndrome are fertile and have been known to have children. However, men with full trisomy 21 are usually infertile.
 
53105. B 20–40%
The prevalence of psychiatric morbidity is 4–10 times that in the general population. Overall, 20–40% have a concurrent psychiatric illness excluding behavioural problems. Proposed reasons for this include:
  • Higher rates in families with a genetic predisposition
  • Higher prevalence of learning disability and mental illness in lower socioeconomic groups
  • Vulnerable population
  • Increased exposure to abuse
  • Multiple traumatic life experiences
  • Lack of early ego-strengthening experiences (e.g. lack of consistent primary caregiver)
  • Lack of protective life experiences, e.g. employment, close confiding relationships
  • Neurochemical abnormalities associated with organic brain abnormalities
  • Increased rates of other risk factors for mental illness, e.g. epilepsy, sensory impairments
  • Syndrome-specific psychiatric risk associated with genetic phenotype, e.g. obsessive–compulsive disorder is associated with Down’s syndrome.
 
106. A Age-specific prevalence varies over time.
The point prevalence of severe intellectual impairment varies between similar cohorts (concurrent age groups) in different communities, e.g. 1.62/1000 children born in 1951–55 in Salford, England, and 7.34/1000 born in 1957 in Amsterdam, The Netherlands. A greater variation is expected in developing countries. Age-specific prevalence varies over time in the same community. A similar pattern of temporal variation is common throughout the developed world, e.g. increases have been the result of decreased early mortality and increased survival associated with better neonatal care. More recently, amniocentesis and abortion programmes have reduced Down’s syndrome prevalence. Prevalence varies by age because of cohort differences from birth and early infancy. Prevalence varies by age and time because survival has increased at all ages. There are usually more men than women at all ages. Studies have tended to show a social class gradient consistent with the known social distribution of morbidity and mortality.
 
107. B Handicap
Impairment is the loss or abnormality of structure or function. Disability means the restriction or lack of ability to perform an activity in the manner or within the range considered normal for a person. Handicap means the disadvantage for an individual that prevents or limits his or her usual performance or role (Table 1.5)
Table 1.5   Assessment of impairment, disability and handicap
Impairment
Established tests of intellectual functioning, e.g. the Wechsler scales
Disability
Established assessments of functioning, e.g. Vineland Adaptive Behaviour Scales, American Adaptive Behaviour Scales, Hampshire Assessment for Living with Others (HALO)
Handicap
Assessments of quality of life and life experiences, e.g. Life Experiences Checklist
 
108. D Offences are broadly similar to those of offenders 54 without learning disability.
Offending in learning disability is:
  • More likely in mild and moderate than in severe learning disability
  • More likely in association with family, social and environmental disadvantage.
Offences are broadly similar to those of offenders without learning disability. There is some evidence that there are increased rates of sex offending and fire-raising in LD individuals.
Learning disability together with antisocial personality features carries a high risk for offending. There has been a recent fall in the use of compulsory admission for people with a learning disability.
Property offences are often committed with a lack of forethought and are opportunistic. People with learning disability are more commonly victims of crime and many of them do not report them to the police. These people put themselves at risk, perhaps due to lack of understanding of social norms and conventions, and chronic low self-esteem. They are more likely to be suggestible at interview and to give false confessions. They are more likely to commit sexual offences, especially against young children and boys. Arson has been consistently associated with learning disability in those referred for hospital treatment.
 
109. A Family members are more likely to have borderline or mild intellectual disability.
Mild or borderline intellectual impairment is more common in families of lower socioeconomic status. There are no dysmorphic features in the affected person as well as in family members.
 
110. A Batten’s disease
This is also called juvenile neuronal ceroid lipofuscinosis (NCL) and Spielmeyer–Vogt–Sjögren–Batten disease. It is an autosomal recessive neurodegenerative disease. It is very rare (2–4 in 100 000 births) but life threatening. Parry’s disease is the adult form of NCL. The earlier the onset of illness, the poorer the prognosis. Early symptoms (visual difficulties, seizures) usually appear around the age of 2–10 years. The muscles have abnormally high tone (myoclonus) with a lack of coordination. There can be dementia or mental impairment with a compromised mental ability. The patients may present with movement disorders such as choreoathetosis, unsteady gait (ataxia) or seizures.
 
111. D Single word reading
The primary skill deficit in dyslexia is difficulty at the level of single word reading. The word-reading problems arise from core cognitive deficits in phonological processing which consist of:
  • Poor phonological awareness of the phonological structure of words in one’s language
  • Difficulties in mastering the alphabetic principle, i.e. connecting written letters (graphemes) with speech sounds (phonemes).
Other prominent impairments seen in most if not all people with dyslexia are poor phonological working memory and impaired rapid autonomic naming. Higher linguistic skills such as syntactic and semantic processing skills are largely intact.
 
112. C PCL-R (Hare)
Actuarial tools attach specific statistical weighting to different variables to assess the risk. The common example in day-to-day life is car insurance quotations, which are compiled using actuarial tools. Other examples include: the Violence Risk Appraisal Guide (VRAG) which is a 12-item actuarial tool for violence risk assessment and includes the PCL-R as a subscale; Violence risk scale; the Static-99; and the SORAG (Sexual Risk Offender Appraisal Guide).
In contrast, structured professional judgement guides consider a number of variables, which will have some application to the assessment of risk in the case under consideration. This allows more flexibility to account for case-specific influences and context, and moves the emphasis from prediction to risk management.
55This question asks for violence risk assessment tools. It does not specify what form of violence, so the candidate must assume that it means physical and sexual violence. Therefore, the ESR-20 should be immediately eliminated, because this is the Estimate of Suicide Risk and a structured professional judgement tool. All the possible answers except the PCL-R (Psychopathy Checklist revised) are structured professional judgement risk assessment tools. The Historical, Clinical, Risk Management-20 (HCR-20) is a well-known tool for formulating violence risk that is commonly used within forensic settings. The Risk for Sexual Violence Protocol (RSVP) and Sexual Violence Risk-20 (SVR-20) are specific to sexual violence.
 
113. B Actus reus
In all offences, it must be proven beyond reasonable doubt that the accused physically did the act, which is known as actus reus. In most offences, it must also be proven that the defendant’s intention or attitude of mind was as required for the crime in question (known as having the necessary mens rea). Therefore, for a crime to be committed the accused should have done the act but must also have intent. Children under the age of 10 in England and Wales, and 8 in Scotland are assumed to be incapable of forming criminal intent.
 
114. A 4%
There has been a review of 62 surveys from prisons in 12 countries; 3.7% of the men had psychotic illnesses. This compares with a figure from the Office for National Statistics (ONS) for the community sample of 0.4%. Prisoners were several times more likely (almost 20 times) to have psychosis and major depression (10–12%) and approximately 10 times more likely to have antisocial personality disorder than the general population. Antisocial personality disorder was exhibited in 47% of male and 21% of female prisoners. Female prisoners show a higher rate of neurotic disorders, substance misuse and self-harm than men. Treatment in the absence of a prisoner’s informed consent is not permitted. However, treatment for mental disorders is permitted under mental health legislation.
 
115. B 1%
Homicide–suicide describes the situation when a person kills someone (often a spouse or relative) and then takes his or her own life. In England and Wales, it is estimated that homicide–suicide accounts for 1% of all homicides. It has been found that a firearm was the most frequently used method for the homicide and suicide, and approximately one in four perpetrators failed in the suicide attempt.
 
116. E Stages 3–4
Offending can occur during sleep, in which case a defence of automatism is appropriate. Sleepwalking occurs during stages 3–4 of slow-wave sleep, not REM sleep when the body is normally paralysed.
It has been suggested that the following factors are necessary for the diagnosis of sleepwalking, especially in relation to assessment of individuals charged under the criminal justice system. The general factors are:
  • Family history: there is a known genetic element to the aetiology of sleepwalking.
  • Childhood onset: sleepwalking commonly begins in childhood, with a lower proportion starting during adolescence.
  • Late-onset of sleepwalking is rare, and more likely after a head injury. Be suspicious if sleepwalking occurs for the first time at the time of the offence.
56Specific factors include:
  • Sleepwalking occurs in stages 3–4 of sleep and should therefore occur within 2 hours of going to sleep.
  • The person should be disoriented on awakening.
  • Any witnesses should report inappropriate automatic behaviours and disorientation on awakening.
  • There should be amnesia for the whole period of sleepwalking.
  • Trigger factors may be present, including drugs, alcohol, excessive fatigue and stress.
  • If the crime is a sexual one, sexual arousal during sleep occurs only during REM sleep and would therefore not occur during sleepwalking.
  • Any memories from before the sleepwalking should be non-narrative and non-dream like.
  • It is unusual for there to be any attempt to conceal a crime committed during sleepwalking.
  • There may have been similar behaviours during previous sleepwalking episode.
  • If the crime appears to be motiveless and out of character, this would give some support to it being committed during sleepwalking.
 
117. B Arson
There is growing evidence that people with Asperger’s syndrome are more likely to commit fire-setting offences than people without the syndrome. Both alcohol and drug misuse and drug offences have been reported in this population, although substance misuse is comparatively rare. Epidemiological studies indicate that people with Asperger’s syndrome do commit sexual offences, but there is evidence that the rates of sexual offending in general, and of child sex offences in particular, are lower among offenders with autistic spectrum disorders.
 
118. B Being unfit to plead can result in a trial of facts.
Fitness to plead is concerned with a defendant’s mental state at the time of trial. The principle of being unfit to plead, which is also known as being under disability, was first established in statute in section 2 of the Criminal Lunatics Act 1800. The tests of disability are derived from the case of a deaf mute (R v Pritchard 1836) and the judgment is interpreted today as follows: to be considered fit to plead, the defendant must have the capacity to:
  • understand the charge
  • instruct a lawyer
  • challenge a juror
  • plead to the charge
  • understand the evidence.
A trial of facts is where a jury, hearing evidence, determines whether the accused ‘did the act or made the omission against him’. It is important to understand that this does not amount to a finding of guilt. If not satisfied on this issue, the jury return a verdict of acquittal.
 
119. A It can be caused by an insulinoma.
This question addresses the candidate’s knowledge of automatism. Automatism is a rare plea in which the defendant claims, at the time of the offence, that he was behaving ‘automatically’ and therefore is not guilty of a crime. The legal definition (Bratty v AG for Northern Ireland 1963) defines automatism, as ‘the state of a person who, though capable of action, is not conscious of what he is doing’. It means an unconscious involuntary action and it is therefore a defence because the mind does not go with what is being done. It is argued that in automatism there is no conscious capacity to control one’s actions and therefore no possibility of mens rea.
There are two forms of automatism:
  1. 57Sane automatism: due to external causes (one-off) such as concussion after a blow to the head or a confusional state after an anaesthetic. Hypoglycaemia after a large insulin injection would also apply. A successful plea of sane automatism results in complete acquittal of the charge.
  2. Insane automatism: due to internal causes of behaviour without voluntary control. Examples include epilepsy, insulinoma (causing episodes of hypoglycaemia) and degenerative brain diseases. A successful plea of insane automatism would lead to the defendant being found not guilty by reason of insanity and therefore the McNaughton rules apply.
 
120. D Polyneuropathy and cirrhosis of the liver
EM Jellinek (1890–1963) was a biostatistician, physiologist and prominent researcher in alcoholism. He developed classification of alcoholism as follows:
  • α alcoholism: this is the earliest stage of alcoholism characterised by psychological dependence on alcohol to relieve physical or psychological pain. He called them ‘problem drinkers’ because their drinking causes personal and social problems. These people can stop drinking if they are determined to do so because they have not lost control and do not have a ‘disease’.
  • β alcoholism: people with this problem drink alcohol heavily, but do not have physical addiction or withdrawal symptoms or a ‘disease’. They do, however, have polyneuropathy and/or cirrhosis of the liver.
  • γ alcoholism: people with this problem have developed tissue tolerance and physical dependence. They have lost control over their drinking and have a ‘disease’.
  • δ alcoholism: they have features of γ alcoholism with an inability to abstain minus loss of control.
  • ε alcoholism: this is a fully fledged disease, which manifests as dipsomania or periodic alcoholism.
 
121. D Three or more manifestations must be present for more than a month.
Three or more manifestations should have occurred together for at least 1 month or occurred together repeatedly within a 12-month period. Tolerance refers to diminished effects with use of the same amount of intake of the substance. A strong desire to take the substance is one of the manifestations that may be present. A physiological withdrawal state should be evident when substance use is stopped or reduced. Continuing to take a substance with clear evidence of harm is one of the manifestations of a dependence syndrome.
 
122. E Partial remission
In patients with alcohol or substance dependence, additional code specifiers may be used to describe the condition further. These code specifiers help us to understand further whether the person is currently abstinent; if so is he in a protective environment such as a rehab setting or is he on treatment? On the other hand, if he is still dependent, this describes the pattern of substance dependence.
 
123. A Cognitive–behavioural therapy
Psychological interventions, such as CBT, behavioural therapies or social network- and environment-based therapies, should be offered to people with harmful drinking and mild alcohol dependence. Such therapies should also be offered after successful withdrawal from moderate-to-severe alcohol dependence. Family therapy should be considered in children and adolescents who misuse alcohol.
 
58124. A GABA agonism
Although the exact mechanism is not known, acamprosate is thought to activate GABA (γ-aminobutyric acid) receptors and block glutamatergic NMDA (N-methyl-d-aspartate) receptors. The mechanism of action involves functional antagonism of ionotropic NMDA receptors. It is noted that direct reactions between acamprosate and NMDA receptors are weak. It is also suggested that there are other possible mechanisms of action. Acamprosate may modulate NMDA receptors via regulatory polyamine sites or directly act on metabotropic glutamate receptors.
 
125. E Specialised cognitive–behavioural therapy
The leading treatment for bulimia nervosa is a specific form of CBT. There is strong research evidence for this treatment and it is recommended in the NICE guidance. Interpersonal psychotherapy offers an evidence-based alternative to CBT and involves about the same amount of therapist contact. However, interpersonal psychotherapy has considerably less empirical support than CBT and takes longer to achieve comparable effects. However, NICE recommends that it should be considered as an alternative to CBT. Both antidepressant drugs and certain self-help programmes have some efficacy and are relatively straightforward to implement. However, as the evidence suggests that a few patients make a full and long-lasting response to either treatment, NICE recommends that these treatments be viewed as possible first steps in management. Fluoxetine 60 mg/day is the antidepressant of choice.
 
126. A It occurs in organic pathological states.
Capgras’ syndrome is a delusional misidentification. The patient believes that an imposter pretending to be that person has replaced someone close to him or her; the abnormality is delusional, not hallucinatory. It is a rare occurrence, which may form part of a schizophrenic illness or affective disorder. Uncommonly it may occur in organic states. In Capgras’ syndrome, there is no outward change in the appearance of the objects and there is no false perception because the patient admits that the double exactly resembles the original.
 
127. A It is a common motivation for homicide.
Morbid jealousy is a disorder of thought content and it may take various forms, e.g. as a delusion, overvalued idea, depressive affect or anxiety state. It is also known as Othello’s syndrome. It occurs in men more often than women. It may be present on its own or as a symptom of schizophrenia, alcohol abuse or cocaine abuse. It is a common cause of domestic abuse and is associated with homicide. The risk of harm is more towards the partner than the rival person alleged to be in the relationship.
 
128. B Hyperaesthesia
Increased intensity of sensations (hyperaesthesia) may be the result of intense emotions or a lowering of the physiological threshold. Examples include seeing roof tiles as brilliant flaming red or hearing the noise of a door closing like a clap of thunder. Anxiety and depressive disorders, as well as hangovers from alcohol and migraine, are all associated with increased sensitivity to noise (hyperacusis). Dysmegalopsia refers to changes in the perceived shapes of objects. There are two types: micropsia, which refers to seeing objects smaller than their actual size, and macropsia, 59 which refers to seeing objects larger than they actually are. Dysmegalopsia results from retinal disease, accommodation and convergence problems, and temporal and parietal lobe lesions. An illusion refers to misrepresentations of stimuli arising from external objects. There are three types of illusions: complete illusions, affect illusions and pareidolia.
 
129. C Couvade’s syndrome
This is an abnormality of self-experience when a man also complains of obstetric symptoms during his partner’s pregnancy and parturition. Gross forms of Couvade’s syndrome are very rare, but minor symptoms in the man directly associated with his partner’s pregnancy are quite common. Symptoms of Couvade’s syndrome in the man occur from the third month of pregnancy onwards (more frequent in months 3 and 9). The symptoms include loss of appetite, toothache, nausea and vomiting (often morning sickness), indigestion, vague abdominal pains, constipation and diarrhoea. Anxiety, tension, insomnia and emotional instability are common complaints and there is a preoccupation with his partner’s condition. The chronological relationship with the partner’s pregnancy is more important for making the diagnosis than the nature of the symptoms. It is not delusional. The man with Couvade’s syndrome does not believe himself to be pregnant. Cotard’s syndrome refers to a condition in which people believe that they are dead or have nihilistic delusions. Autoscopy is also known as phantom mirror image. It is the experience of seeing oneself and recognising that it is oneself. It can occur in healthy people when they are tired or emotionally exhausted. Autoscopy can also occur in patients with schizophrenia and organic brain disorders.
 
130. A Cognitive–analytic therapy
  • Factors favouring integrative, structured brief therapy (e.g. cognitive–analytic therapy or CAT): when the issues are more complex and intertwined and related to particular stressors, typically definable pattern of relationship issues, and ongoing and repeated self-limiting but avoided themes.
  • Factors favouring short-term dynamic therapy: when the problems are clear and definable, and conflicts are more focal than general. The aim of the therapy is to achieve limited goals in a limited time and the problems are more ‘oedipal’ in nature, e.g. competition anxiety, authority and power problems, conflict in triangular relationships and anxiety in sexual relationships. There should be enough trust between the patient and the therapist to help in tolerating the frustration that might surface during the therapy.
  • Factors favouring CBT: abnormal thought patterns (cognition) or behaviours that are maintaining dysfunctional patterns in the person’s life. The patient is aware of the problem and doesn’t have any overwhelming relationship issues that may interfere in establishing a therapeutic relationship.
  • Factors for long-term individual therapy: there is a lack of clarity about the problems, the conflicts are not clearly definable, there are severe difficult personality issues, and the goals of the therapy can be defined in terms of general interpersonal function, i.e. control or intimacy.
60
 
Answers: EMIs
 
131. A Correlation coefficient
This indicates the degree to which two measures are related although it does not explain how.
 
132. B Kendall’s correlational coefficient
This is a correlational coefficient for two categorical or non-parametrically (non-normally) distributed variables.
 
133. C Logistic regression
This is used when the dependent variable is of a dichotomous or binary type (e.g. yes or no) whereas the independent variable can be of any type.
 
134. D Multiple analysis of co-variance
This is used with multiple dependent and independent variables.
 
135. E Multiple analysis of variance
This is used with multiple dependent variables.
 
136. B Blinding
This means that both the participants and the investigators are unaware of the information about which group the participants are allocated to. This is to avoid any bias that could arise in the study.
 
137. E Recall bias
When compared with healthy control participants, individuals with the disorder are likely to have increased health awareness and greater understanding of the disease process. ‘Response priming’ can occur in a study with participants who have chronic conditions. The ability to recall past events accurately varies from person to person. This is a problem in retrospective studies. Multiple sources of information help corroborate data.
 
138. C Matching
In this scenario, controlled participants are selected based on certain characteristics (usually potential confounding factors) that they have in common with experimental participants. This is done to ensure that confounders are distributed in both groups.
 
61139. F Restriction
This is a way of achieving reduction of confounding factors. In this study, they were focusing on non-cannabis-related factors in schizophrenia.
 
140. G Stratification
In this study, economics status was a confounding factor, hence the data stratification.
 
141. J Synaesthesia
The word ‘synaesthesia’ originated from ‘syn’, a Greek word meaning together and ‘aesthesia’ meaning sensation. Psychomimetic drugs such as LSD can induce multiple perceptual changes at the same time, including changes in spatial orientation, perception of movement, colour appreciation, visual illusions and hallucinations at once. Synaesthesia occurs when a stimulus image in one sensory modality is perceived as an image in another sensory modality. It is not an image in the mind’s eye; the experience is real and is projected outside the individual. He can describe the colour, shape and taste of someone’s voice.
 
142. B Charles Bonnet syndrome
This is a syndrome (phantom visual images) in which the patient experiences complex visual hallucinations that are not associated with any other psychiatric abnormality. It usually occurs with loss of vision. Although more common in elderly people, it can occur at any age and follows central or peripheral vision loss. It can last from days to years and patient can see human images, animals, buildings, and static or moving scenery.
 
143. K Temporal lobe epilepsy (TLE)
Auditory and visual hallucinations can occur synchronously in pathological organic states of TLE.
Children with complex febrile seizures lasting >15 min have focal features, or recur within 24 hours. Relationship of TLE with childhood febrile convulsions is controversial. Other causes of TLE include head trauma resulting in encephalomalacia or cortical scarring, infections such as meningitis, vascular malformations such as arteriovenous malformations, traumatic delivery such as forceps deliveries, hamartomas in the brain, meningiomas, etc., cryptogenic (of obscure origin) and rarely idiopathic (genetic).
 
144. D Delirium tremens
This is an acute organic condition due to alcohol withdrawal, coloured by gross changes in perception mood and conscious state. The perceptual disturbance usually starts with pareidolic or affective illusions followed by visual and the haptic lilliputian hallucinations. These experiences are in a constant state of flux rapidly changing from one form to the other, causing confusion and terror in the patients. Illusions are usually mixed with hallucinations and are very frightening. There is a high level of suggestibility in patients at these times, which can lead to abnormal visual experience as a result of suggestibility.
 
62145. B CATIE
Thus is an acronym for ‘Clinical Antipsychotic Trials of Intervention Effectiveness’. It assessed the effectiveness of antipsychotic medication used in the treatment of schizophrenia and compared older antipsychotics with newer antipsychotics. The antipsychotics used in the trial were perphenazine (first generation) and second-generation drugs: olanzapine, quetiapine, risperidone and ziprasidone. It was a double-blind randomized trial. The follow-up period was for 18 months. The primary measure was to check the duration for which a patient remained on medication.
 
146. I SOHO
Schizophrenia Outpatient Health Outcomes study (SOHO) is an observational study of outpatients who received antipsychotic medication for the treatment of schizophrenia. Ten European countries participated in the study. Data on more than 9000 patients was analysed in the study.
 
147. D DEPRES
This was the first pan-European survey of depression in the community in 1990. In the second phase of the study six depressed patient types were identified with clearly differentiated profiles based on cluster analysis of the data. It was set up to examine depression across six European countries including the UK. The most commonly experienced symptoms of depression were low mood (76%), tiredness (73%) and sleep problems (63%). SmithKline Beecham Pharmaceuticals funded the study. The study demonstrated that 17% of the general population has depression. More patients on SSRIs felt like normal self than those on tricyclic antidepressants.
 
148. E ESEMeD
This was conducted in six studies across Europe, namely Belgium, France, Germany, Italy, the Netherlands and Spain. It was initially named the European Study of the Epidemiology of Mental Disorders/Mental Health Disability (ESEMeD/MHEDEA). The age range of participants was ≥18 years. It was a cross-sectional study. Patients receiving medication and/or psychological therapies were considered in the study. Assessments took place in the community with the help of computer-assisted techniques. Questionnaires were based on CIDI (Composite International Diagnostic Interview).
 
149. H POTS
This was conducted in three academic centres across the USA. It was conducted to evaluate the efficacy of CBT alone, sertraline alone, and CBT and sertraline combined. The duration of treatment was 12 weeks and participants were recruited between September 1997 and December 2002. The findings of the study suggested that children and adolescents should begin treatment with either a combination of CBT and a SSRI or CBT alone.
 
150. C CUtLASS 1
This study essentially compared first-generation antipsychotics with second-generation antipsychotics (except clozapine) in the treatment of schizophrenia. The outcome measure used was quality of life. The results showed no significant improvement in the outcome in any of the groups. Interestingly patients on first-generation antipsychotics showed better quality of life compared with second-generation antipsychotics Table 1.6
63
Table 1.6   Abbreviations of studies
AESOP
Aetiology and Ethnicity of Schizophrenia and Other Psychoses
CAFÉ
Comparison of Atypicals in First Episode of psychosis
CATIE
Clinical Antipsychotic Trials of Intervention Effectiveness
CUtLASS
Cost Utility of the Latest Antipsychotic Drugs in Schizophrenia Study
DEPRES
Depression Research in European Society
DOSMD
Determinants of Outcome of Severe Mental Disorders
ESEMeD
European Study of the Epidemiology of Mental Disorders
EUFEST
European First Episode Schizophrenia Trial
IPSS
International Pilot Study of Schizophrenia
MTA
Multimodal Treatment of ADHD
NEMESIS
Netherlands Mental Health Survey and Incidence Study
POTS
Pediatric OCD Treatment Study
SOHO
Schizophrenia outpatient health outcomes
STAR*D
Sequenced Treatment Alternatives to Relieve Depression (STAR*D)
TADS
Treatment for Adolescents with Depression Study
TEOSS
Treatment of Early Onset Schizophrenia Spectrum disorders
TORDIA
Treatment Of SSRI Resistant Depression in Adolescents
 
151. D Point prevalence
This refers to the proportion of people affected by the condition or disease in study in a defined population at a specified period of time.
 
152. A Cumulative incidence
This measures the proportion of people developing a disease or disorder in a specified population over a period of time. Hence, as the time period of study increases, cumulative incidence would also increase. It ranges from 0 to 1. The time period is also mentioned.
 
153. F Standardised mortality rate
The mortality rate is the incidence rate that expresses the risk of death in a population over a time period. It is often useful to have a single measure of mortality for a given population, which allows for the make-up of that population to compare different populations on one measure rather than several measures. An adjustment or standardisation process achieves this.
Life-time prevalence refers to the proportion of people who had a given condition or disease at least once in their lifetime in a given population.
Morbidity rate refers to the rate measuring the occurrence of disease in a specified population. Morbidity rates include incidence and prevalence rates.
Standardised mortality ratio: ratio of observed mortality rate to the expected mortality rate in the population.
Period prevalence: the proportion of the population that has the disease in a given period.
 
64154. B Play the winner
In this approach, the first participant is allocated by simple randomisation, and thereafter every subsequent allocation is based on the success or failure of the immediate preceding participant.
 
155. F Randomisation permutated blocks
This requires randomisation in a group of, say, four, six, etc. to ensure that the numbers are equal in the two groups.
 
156. D Stratified randomisation
This divides the participants into homogeneous subgroups before sampling. The strata should be mutually exclusive, i.e. every element in the population must be assigned to only one stratum/group. It helps to select representative samples by reducing sampling error. It allocates participants on the basis of the same variables, e.g. prognosis, to ensure that they are evenly distributed within the study. However, it requires an additional schedule for each stratum. Stratification can produce a weighted mean that has less variability compared with the arithmetic mean of a random sample.
 
157. E Randomised consent method
This is used occasionally to lessen the effect of some patients refusing to participate in a study.
 
158. C Each participant received both the intervention and the treatment, G Separated by a period of no treatment
Cross-over trials are a real option in relatively rare diseases in which the number of available participants may not permit a randomised, parallel-group, controlled trial. The participants act as their own controls. These trials are useful in psychology, education, pharmaceutical science and medicine (diagnosis, treatment and prevention of disease). They offer two advantages over non-crossover studies: the influence of confounding co-variates is reduced and they are statistically efficient because fewer participants are required. They have several disadvantages: historical controls, ‘order’, ‘learning’, and ‘carry-over’ effects. An n-of-1 trial is a special example of a cross-over trial.
 
159. B Each group receives a different treatment with both groups being entered at the same time, E Results are analysed by comparing groups
Parallel-group comparison is a part of controlled trials that is necessary to evaluate a new treatment against a placebo or gold standard treatment. This comparison sometimes overestimates the therapeutic benefits.
 
160. D Participants are assessed before and after an intervention, F Results are analysed in terms of differences between participant pairs
In this type of study design, the participants are offered two alternatives and then asked to indicate to the researchers which alternative they like most. Paired comparison can be included in uncontrolled, controlled and randomised controlled trials of new treatments.
 
65161. A Assumes a poor outcome for drop-outs, E Differences in the drop-out rates and the timing of these drop-outs influence the estimation of treatment
Last observation carried forward (LOCF) is an analytical method used to evaluate a missing value in a clinical trial from the last measurement in an individual participant. Participants who drop out for various reasons, such as side effects or ineffective treatment, are considered treatment failures. LOCF may therefore under- or overestimate the benefits of treatment.
 
162. D Data on all patients entering a trial should be analysed with respect to the groups to which they were originally randomised. regardless of whether or not they received treatment, E Differences in the drop-out rates and the timing of these drop-outs influence the estimation of treatment
Intention-to-treat analysis is a method to analyse the data generated in the course of a clinical trial according to assigned groups. Trial drop-outs are regarded as treatment failures and cannot be ignored in the analysis of data. All clinical trials should conduct intention-to-treat analysis to avoid overestimating the benefits of treatment. Generally, a continuous measure such as a symptom severity score gives more statistical power than a dichotomous or categorical measure. For a continuous measure, LOCF is used as the final measure.
 
163. C Data should be analysed as unit of randomisation, F Interventions are directed at groups rather than individual participants
Cluster trials are a special type of RCTs in which interventions are directed at groups rather than individuals. They are commonly used in evaluating more global aspects of health services than one particular treatment, or where allocation of participants is not practicable. They require analysis in clusters, i.e. unit of randomisation rather than individuals, which needs a lot of clusters to detect a statistically significant effect.
 
164. B Investigations, F Staff salaries
Health economics can be applied to various aspects of healthcare such as treatment. They can help to make decisions about value for money by comparing the effects of competing interventions. An economic analysis requires consideration of the underlying research approach and relevant inputs: direct costs (medical expenses), and indirect costs and inputs. Direct costs are the resources consumed by the programme rather than an alternative.
 
165. C Pain and suffering, E Social stigma
Intangible costs are immeasurable human costs such as pain, suffering stigma, etc.
 
166. G Value of ‘unpaid work’, H Work days lost
Indirect costs are productivity gains and losses with the focus on patient time consumed or freed up by healthcare programmes. They are also referred to as overhead costs, e.g. electricity, gas, rental, capital costs.
 
66167. F Health STAR
Health STAR (Health – Services Technology, Administration and Research) contains materials specifically selected with a focus on health services, research and non-clinical aspects of healthcare delivery (e.g. healthcare administration, economics planning and policy).
 
168. B CINAHL
The Cumulative Index to Nursing and Allied Health Literature (CINAHL) is a database for nursing literature.
 
169. D EMBASE
This is a database of pharmaceutical and biomedical literature produced by Elsevier. It concentrates on European sources and drug-related literature. The Cochrane Library is a collection of databases, most of which are solely databases of systematic reviews of healthcare interventions. It provides a quick and focused way of locating RCTs. MEDLINE is a major source of published biomedical scientific literature produced by the National Library of Medicine in the USA. It contains thousands of pieces of published data.
 
170. D Examples include case reports and series, audits, cross-sectional surveys and qualitative study, F Generally conducted without a control group, H Suitable for hypothesis generation
Descriptive studies are generally conducted without a control group, and are suitable only for generating hypotheses. Examples include case reports, case series, audits, cross-sectional surveys of psychiatric morbidity and qualitative studies.
 
171. C Examples include case–control and cohort studies, E Generally compare two groups, I Suitable for hypothesis testing
Analytical studies include two groups of participants: cases and controls can be healthy participants or those with a different disease. They are suitable for testing a hypothesis. Examples are case–control studies and cohort studies.
 
172. A Causation can be inferred, B Examples are controlled clinical trial and economic analyses, systematic reviews and their meta-analyses, G Something is given or done in the experimental group but not in the control group
Experimental studies include ‘experimental’ and control groups. Any resulting differences in outcome are measured, from which causation may be inferred. Examples of experimental studies are controlled clinical trial and economic analyses, systematic reviews and their meta-analysis.
 
173. G Single blinding
This helps to reduce potential biases. In a single-blind design, the participants are not aware of the treatment that they are receiving, i.e. placebo or active drug.
 
67174. A Double blinding
In a double-blind study design, both the participants and the investigators are not aware of the treatment that the participants are receiving. This design helps to reduce investigators’ bias.
 
175. G Triple blinding
In this design, the analysis of outcome is carried out by independent investigators who are blind to the treatment given. Both the investigators and the participants are unaware of the treatment that the participants are receiving.
 
176. D Recall bias
This is introduced by selective memory of participants or informants, who are likely to ‘search after meaning’ and identify possible exposures. It is also sometimes called ‘rumination bias’, e.g. depressed patients may be more likely to remember adverse life events. Sometimes, participants may alter their responses in the direction that they perceive is desired by the investigators. This is known as ‘obsequiousness bias’.
 
177. B Incidence risk
This is the number of new cases of a disease over a period of time out of the total population at risk. It is also called cumulative incidence because the number developing disease may change over time through death or loss to follow-up.
 
178. A Incidence density
This is the new cases of a disease out of the total person-time of observation. It is also called the incidence rate. It provides varying person-years of observation, and is more reliable for quantifying the risk of developing a disease over a certain period of time.
 
179. E Period prevalence
This includes point prevalence (number of new cases in a population at a particular point in time) and new cases over a period of time.
 
180. G Standardised mortality ratio
This is calculated as the ratio of observed to expected deaths. It can be analysed by parametric or non-parametric statistics depending on their distribution. The mortality rate is an incidence rate that expresses the risk of death in a population over a period of time. It is useful to have a single measure of mortality for a population to allow meaningful comparison between different populations.
 
181. E Response set
Some individuals have the tendency to either agree or disagree. They may not reveal their true opinion in a research study. It is human nature to alter one’s behaviour when we know that we are being observed. This is called attention bias.
 
182. F Social acceptability
Most people have a tendency to minimise any perceived deviation from the norm, e.g. some 68 relative or the patient minimises the history of substance misuse or other psychiatric symptoms. Some people simply avoid the question altogether, and this is called ‘undesirability bias’.
 
183. D Hawthorne’s effect
This refers to the non-specific effects caused by the knowledge that the research participants have that they are participating in the experimental study. The confounding can be reduced by randomisation, restriction (selecting only participants who have a particular range of values of a potential confounder, e.g. education, social class) and matching. Statistical control of confounding involves two methods: stratified analysis and multivariate analysis using regression.
Bias towards the centre: participants avoid extreme responses and stick to the middle.
Halo effect: this is the observer’s error where one judgement is carried over from another. It is a psychological tendency of people to judge others on the basis of a single trait of which they approve but they also conclude that the person must have other attractive traits.
Extreme responding: the participants always agree or disagree with the questions.
 
184. E Orbitofrontal tumours
Impulsivity with behavioural disinhibition is due to orbitofrontal tumours. There are personality changes, mood lability, irritability, lack of insight with poor judgement, social inappropriateness with immoral language, inappropriate sexuality and tactless gaiety.
 
185. C Dorsolateral prefrontal convexity tumours
There is apathy and lack of spontaneity, along with abulia, reduced planning ability, psychomotor retardation, impaired concentration, attention and motor impersistence. The symptoms mimic depressive illness.
 
186. A Anterior cingulate tumours
They cause akinetic mutism.
 
187. H RD Laing
He was one of the most prominent figures in the anti-psychiatry movement. He challenged the diagnostic labels and the medical model of mental illnesses. He was a Scottish psychiatrist who wrote extensively on mental illness, in particular the experience of psychosis.
 
188. A Benedict Morel
In 1860, he observed young men being affected by a mental disorder that eventually led to reduced mental functioning and disability. He used the term ‘démence precoce’ to describe it. As time progressed, it was realised that démence precoce didn’t necessarily affect only young adults or lead to mental deterioration. In 1908, Eugene Bleuler coined the term ‘schizophrenia’ for this recognised disorder.
 
189. F Karl Ludwig Kahlbaum
In 1874, he first described catatonia as a disorder of unusual motor symptoms. He studied and described the natural course of the symptoms of catatonia as a disorder. Motor immobility (holding rigid postures), motor hyperactivity, mutism, etc. are characteristic features.
 
69190. E John Cade
Lithium was initially introduced to treat bladder stones and gout in the mid-nineteenth century. In the later part of that century, it was shown to be helpful in depression. In the 1940s, lithium chloride was unsuccessfully used to replace sodium chloride for hypertensive patients. John Cade reported its successful use in mania and, later on in the 1950s and 1960s, Mogans Schou demonstrated short-term prophylactic efficacy in bipolar 1 disorder.
 
191. D Disqualifying the positive
See Table 1.7 for explanation.
 
192. I Selective abstraction
See Table 1.7 for explanation.
 
193. B Catastrophising
See Table 1.7 for explanation.
Table 1.7   Cognitive distortions
Arbitrary inference
Jumping to conclusions
A person quickly draws a conclusion without the requisite evidence
Interpreting situations incorrectly by drawing unjustified conclusions
Catastrophising
Magnification
Blowing things out of proportion
Focusing on the worst possible outcome
Common in panic
Dichotomous thinking
All or nothing thinking
Black or white thinking
Failure to see the middle ground or grey area
Disqualifying the positive
Failing to acknowledge positive events or aspects of a situation
Common in depression
Magical thinking
Common in obsessive–compulsive disorder
For example, because I thought about death, something bad will happen to someone I love
Minimisation
Giving less importance to a situation/issue than it deserves
Overgeneralisation
Unfairly generalising an occurrence from one situation to other situations to which it may not apply
Personalisation
Attributing outcomes or events to oneself when external factors are also of importance
Selective abstraction
Detail is taken out of context and believed while everything else is ignored
Acknowledgement of one aspect of a situation while ignoring others
Picking out a single negative and dwelling on it while ignoring the rest
For example, depressive negative selection
 
70194. E Moderate depression
According to NICE guidelines, young people should not be offered medication for mild depression. For moderate-to-severe depression, fluoxetine should be offered/considered along with psychological therapies. Sertraline and citalopram are second-line drugs. Paroxetine, venlafaxine, tricyclic antidepressants and St John’s wort should not be prescribed.
 
195. G Severe ADHD without co-morbidity
According to NICE guidelines, drug treatment should be offered in school-age children and young people with severe ADHD. Methylphenidate should be considered for ADHD with and without comorbid conditions such as conduct disorder. Atomoxetine should be considered if methylphenidate has been ineffective or if the young person is intolerant of the drug.
 
196. A Acute mania
For treatment of children and young people with acute mania, medication management is similar to that in adults except that they should be started at low doses. Height, weight and prolactin levels should be checked at the initial presentation. Olanzapine, risperidone and quetiapine are the options available. It should be noted that lithium is the only drug with current marketing authorisation in the UK for bipolar disorder in patients aged between 12 and 18 years.
 
197. E Lesch–Nyhan syndrome
Children with the Lesch–Nyhan syndrome show verbal and physical aggression. However, a more characteristic problem, which is resistant to management, is self-injurious behaviour; this affects >85% of these children. Most prominent self-injurious behaviours include biting the lips, the inside of the mouth and the fingers. Other common self-injurious behaviours include thumping of the ears and face, and hitting the head against objects. Although in some cases these behaviours start in adolescence, in most cases they start between the ages of 2 and 3½ years. It is reported in some cases that self-injurious behaviours reduce in frequency and severity after the age of 10.
 
198. F Prader–Willi syndrome
The behavioural phenotype associated with the Prader–Willi syndrome has been described consisting of abnormalities in speech, sleep and behaviour, along with a specific pattern of cognitive impairment in which individuals show relatively intact visuospatial skills associated with reduced short-term memory and greater loss of memory over time.
Speech abnormalities, including articulation disorder, abnormal syntax among others, have been described.
Sleep abnormalities, including excessive daytime sleepiness, initial insomnia, sleep-onset REM and hyperventilation during REM sleep, have been reported.
A behaviour syndrome consisting of motor slowness, skin picking, sleepiness, ritualistic behaviour, impulsive talk and stubbornness has been described.
 
199. A Angelman’s syndrome
This is also known as the ‘happy puppet syndrome’. Angelman’s syndrome is characterised by severe learning disability, jerky limb movements associated with abnormal gait and inappropriate bouts of laughter. Most affected children lack speech. Other characteristic clinical features include epilepsy (about 86%) and/or and abnormal EEG, tongue thrusting, hand flapping and mouthing behaviour.
71Typical facial features associated with this syndrome consist of long face and prominent jaw, a wide mouth and widely spaced teeth, thin upper lip, midfacial hypoplasia, deep-set blue eyes, blonde hair, flat occiput and microcephaly.
 
200. G Rett’s syndrome
In a postal questionnaire survey of 107 families who rated behaviours among their children with Rett’s syndrome, episodes of low mood were reported in 70%, brief episodes of mood changes in 67% and sustained mood changes in 9%. They also reported episodes of anxiety with hyperventilation, screaming, self-injury, a frightened expression and general distress, precipitated by sudden noises, certain music, strange people or places, changes in routine and excessive environmental activity. Self-injurious behaviour may occur in 40–50% of affected girls and is often associated with abnormal hand movements including biting and chewing of the fingers and hands. More severe self-injury may involve hand-to-hand banging, hand-to-object banging, hair pulling, scratching and head banging. Sleep problems, including laughing at night, may affect up to 74% of cases. Many girls also manifest autistic features and between 59 and 72% develop epileptic seizures by age 5.72